Guyton and Hall Physiology 04








































































































































submerged in inert oil. An oxygen value of 10 mm Hg is recorded. The

distal duodenum at the same location is then treated with the

vasodilator, adenosine. Which value of oxygen is most likely in the tip

of the villus within 2 minutes after treatment with adenosine (in mm

Hg)?

A) 0

B) 5

C) 7

D) 10

E) 12

57. One of the following hormones can stimulate growth of the intestinal

mucosa, and two other hormones can stimulate pancreatic growth.

Which three hormones are these?

Gastrin Secretin Cholecystokinin GIP Motilin

A) No Yes Yes Yes No

B) Yes No Yes No Yes

C) Yes No Yes Yes No

D) Yes No Yes Yes No

E) Yes Yes Yes No No

58. A 65-year-old man eats a healthy meal consisting of 30%

carbohydrates, 20% fats, and 50% proteins. Approximately 40 minutes

later, the ileocecal sphincter relaxes, and chyme moves into the cecum.

Gastric distention leads to relaxation of the ileocecal sphincter by way

of which reflex?

A) Enterogastric

B) Gastroileal

C) Gastrocolic

D) Intestino-intestinal

E) Rectosphincteric

59. The gastric mucosal barrier has a physiological and an anatomical basis

to prevent hydrogen ion accumulation in the mucosa. Some factors are

known to strengthen the integrity of the gastric mucosal barrier,

whereas other factors can weaken the barrier. Which factors strengthen

or weaken the barrier?

Bile Salts Mucous Aspirin NSAIDs Gastrin Ethanol

A) Strengthen Strengthen Weaken Weaken Strengthen Strengthen

B) Strengthen Strengthen Weaken Weaken Weaken Strengthen

C) Weaken Strengthen Strengthen Weaken Strengthen Weaken

D) Weaken Strengthen Weaken Weaken Strengthen Weaken

E) Weaken Weaken Weaken Streng-then Strengthen Weaken

60. The assimilation of fats includes (1) micelle formation, (2) secretion of

chylomicrons, (3) emulsification of fat, and (4) absorption of fat by

enterocytes. Which sequence best describes the correct temporal order

of these events?

A) 4, 3, 2, 1

B) 3, 1, 4, 2

C) 3, 4, 1, 2

D) 2, 1, 4, 3

E) 4, 2, 1, 3

F) 2, 4, 1, 3

G) 1, 2, 3, 4

H) 1, 3, 2, 4

61. A 62-year-old man with dyspepsia and a history of chronic gastric

ulcer has abdominal pain. Endoscopy shows a large ulcer in the

proximal gastric body. Biopsies are positive for H. pylori. Which

substances are used clinically for treatment of gastric ulcers of various

etiologies?

Antibiotics NSAIDs H2 Blockers Proton Pump Inhibitors

A) No No Yes Yes

B) Yes No No Yes

C) Yes No Yes Yes

D) Yes Yes Yes Yes

E) No Yes Yes Yes

62. Cystic fibrosis (CF) is an inherited disorder of the exocrine glands

affecting children and young people. Mucus in the exocrine glands

becomes thick and sticky and eventually blocks the ducts of these

glands (especially in the pancreas, lungs, and liver), forming cysts. A

primary disruption in the transfer of which ion across cell membranes

occurs in CF, leading to decreased secretion of fluid?

A) Calcium

B) Chloride

C) Phosphate

D) Potassium

E) Sodium

63. A 43-year-old man presents with abdominal pain and hematemesis. An

abdominal examination was relatively benign, and abdominal

radiographs were suggestive of a perforated viscus. Endoscopy

revealed a chronically perforated gastric ulcer, through which the liver

was visible. Which mechanism is a forerunner to gastric ulcer

formation?

A) Back-leak of hydrogen ions

B) Mucus secretion

C) Proton pump inhibition

D) Tight junctions between cells

E) Vagotomy

64. A 12-year-old girl consumes a glass of milk and two cookies. Her lower

esophageal sphincter (LES) and fundus relax while the food is still in

the esophagus. Which substance is most likely to cause relaxation of the

LES and fundus in this girl?

A) Gastrin

B). Histamine

C) Motilin

D) Nitric oxide

E) Norepinephrine

65. Mass movements can be stimulated after a meal by distention of the

stomach (gastrocolic reflex) and distention of the duodenum

(duodenocolic reflex). Mass movements often lead to which of the

following?

A) Bowel movements

B) Gastric movements

C) Haustrations

D) Esophageal contractions

E) Pharyngeal peristalsis

66. A 45-year-old woman with type 1 diabetes has an early feeling of

fullness when eating. She is often nauseous after a meal and vomits

about once each week after eating. Glucose-induced damage to which

structure is most likely to explain her gastrointestinal problem?

A) Celiac ganglia

B) Enteric nervous system

C) Esophagus

D) Stomach

E) Vagus nerve

67. Which stimulus–mediator pair normally inhibits gastrin release?

Stimulus Mediator

A) Acid CCK

B) Acid GIP

C) Acid Somatostatin

D) Fay acid Motilin

E) Fay acid Somatostatin

68. A 55-year-old man consumes a meal consisting of 20% fat, 50%

carbohydrate, and 30% protein. The following gastrointestinal

hormones are released at various times during the next 6 hours: gastrin,

secretin, motilin, glucose-dependent insulinotropic peptide, and

cholecystokinin. Which structure is most likely to release all five

hormones in this man?

A) Antrum

B) Colon

C) Duodenum

D) Esophagus

E) Ileum

69. A 79-year-old man has a cerebrovascular accident (stroke) in the

medulla and pons that eliminates all vagal output to the gastrointestinal

tract. Which function is most likely to be eliminated in this man?

A) Gastric acid secretion

B) Gastrin release

C) Pancreatic bicarbonate secretion

D) Primary esophageal peristalsis

E) Secondary esophageal peristalsis

70. A 74-year-old man with hematemesis and melena is diagnosed with a

duodenal ulcer. Which of the following is most likely in this patient?

Parietal Cell Density Acid Secretion Plasma Gastrin

A) Decreased Decreased Decreased

B) Decreased Increased Decreased

C)

Increased Decreased Increased

D)

Increased Increased Decreased

E)

Increased Increased Increased

71. A 61-year-old man with upper abdominal pain and blood in the stool

takes NSAIDs for the pain and washes it down with vodka.

Pentagastrin administration produced lower than predicted levels of

gastric acid secretion. Secretion of which substance is most likely to be

diminished in this patient with gastritis?

A) Intrinsic factor

B) Ptyalin

C) Rennin

D) Saliva

E) Trypsin

72. Gastric acid is secreted when a meal is consumed. Which factors have a

direct action on the parietal cell to stimulate acid secretion?

Gastrin Somatostatin Acetylcholine Histamine

A) No No Yes Yes

B) Yes No No Yes

C) Yes No Yes Yes

D) Yes Yes Yes Yes

E) Yes Yes No Yes

73. A 37-year-old woman adds high-fiber wheat and bran foods to her diet

to reduce her serum cholesterol levels. She had avoided eating foods

containing wheat or rye since she was a child because her mother said

they would make her sick. The woman loses 25 lb on her new diet but

has frequent stomach cramps, gas, and diarrhea. She has also become

weaker, finding it difficult to complete her morning walks. What is most

likely to be increased in this woman?

A) Blood hemoglobin concentration

B) Carbohydrate absorption

C) Fecal fat

D) Protein absorption

E) Serum calcium

74. A newborn boy does not pass meconium within 48 hours of delivery.

His abdomen is distended, and he begins vomiting. A suction biopsy of

a distally narrowed segment of the colon shows a lack of ganglionic

nerve cells. This newborn is at risk for developing which condition?

A) Achalasia

B) Enterocolitis

C) Halitosis

D) Pancreatitis

E) Peptic ulcer

75. A 41-year-old obese woman with a history of gallstones is admied to

the emergency department because of excruciating pain in the upper

right quadrant. The woman is jaundiced, and a radiograph suggests

obstruction of the common bile duct. Which values of direct and

indirect bilirubin are most likely to be present in the plasma of this

woman (in milligrams per deciliter)?

Direct Indirect

A) 1.0 1.3

B) 2.3 2.4

C) 5.0 1.7

D) 1.8 6.4

E) 6.8 7.5

76. Which mechanism for transport of substances across the luminal cell

membrane of a duodenal enterocyte is present in newborns and infants

but not in adults?

A) Endocytosis

B) Facilitated diffusion

C) Passive diffusion

D) Primary active transport

E) Secondary active transport

77. Cystic fibrosis is the most common cause of pancreatitis in children.

Which option best explains the mechanism of cystic fibrosis-induced

pancreatitis?

A) Activation of enterokinase

B) Activation of trypsin inhibitor

C) Autodigestion of pancreas

D) Excessive secretion of CCK

E) Gallstone obstruction

Answers

1. B) Gastrin is secreted mainly by the G-cells of the antrum of the stomach.

The primary actions of gastrin are (1) stimulation of gastric acid secretion

and (2) stimulation of mucosal growth throughout the gastrointestinal

tract.

TMP14 p. 792

2. D) Motilin is secreted by the duodenum and jejunum during fasting, and

the only known function of this hormone is to increase gastrointestinal

motility. Motilin is released cyclically and stimulates waves of

gastrointestinal motility called interdigestive myoelectric complexes (or

migrating motility complexes) that move through the stomach and small

intestine every 90 minutes in a person who has fasted. The purpose of

these interdigestive myoelectric complexes is to remove food residue from

the intestine, which lowers bacterial growth. Hence, interdigestive

myoelectric complexes are also called housekeeping contractions.

TMP14 pp. 792–793

3. B) Gastrin is secreted in response to vagal stimulation as well as stimuli

associated with ingestion of a meal, such as distention of the stomach and

the breakdown products of proteins. However, fat does not cause the

release of gastrin as it does with other gastrointestinal hormones.

TMP14 p. 792

4. D) Under normal conditions, the blood flow of the gastrointestinal tract is

directly related to the level of local activity. For instance, after a meal, the

motor activity, secretory activity, and absorptive activity all increase;

likewise, the blood flow increases greatly. This increase in blood flow is

caused by vasodilation of intestinal blood vessels. The increase in cardiac

output associated with consuming a large meal is causing ischemic chest

pain in this man.

TMP14 pp. 794–795

5. D) Sodium ions are actively reabsorbed from the salivary ducts, and

potassium ions are actively secreted in exchange for the sodium. When

salivary flow is elevated, each unit portion of saliva spends less time in the

salivary ducts. Hence, there is less time for absorption of sodium

(increased sodium concentration) and less time for potassium secretion

(decreased potassium concentration).

TMP14 p. 808

6. D) This student has temporary lactose intolerance resulting from the

temporary loss of the enzyme lactase. Lactase is a brush border enzyme; its

production can be temporarily depressed following an intestinal viral

infection (viral gastroenteritis).

TMP14 p. 824

7. F) The presence of acid, fat, protein breakdown products, hyperosmotic or

hypo-osmotic fluids, or any irritating factor in the upper small intestine

causes release of several intestinal hormones. One of these hormones is

secretin, which is especially important for stimulating pancreatic

bicarbonate secretion. However, secretin opposes gastric acid secretion.

Three other hormones—glucose-dependent insulinotropic peptide

(formerly called gastric inhibitory peptide), vasoactive intestinal

polypeptide, and somatostatin—also have slight to moderate effects in

inhibiting gastric acid secretion.

TMP14 pp. 792, 814

8. E) The same cells that secrete proteolytic enzymes into the acini of the

pancreas simultaneously secrete another substance called trypsin inhibitor;

this substance prevents activation of trypsin both inside the secretory cells

and in the acini and ducts of the pancreas. When the pancreas becomes

severely damaged or when a duct becomes blocked, large quantities of

pancreatic juice can become pooled in the pancreas. Under these

conditions, the effect of trypsin inhibitor is often overwhelmed, in which

case the pancreatic secretions rapidly become activated and can literally

digest large portions of the pancreas within a few hours, giving rise to the

condition called acute pancreatitis. The obese woman described in this

question has gallstone obstruction at the sphincter of Oddi, which causes

pancreatic juice to pool within the pancreas, leading to autodigestion of the

pancreas.

TMP14 pp. 815, 835

9. C) Fructose transport does not occur by the sodium co-transport

mechanism used for glucose and galactose absorption. Instead, fructose is

transported by facilitated diffusion all the way through the intestinal

epithelium and is not coupled with sodium transport.

TMP14 p. 831

10. A) When liver parenchymal cells are destroyed, they are replaced with

fibrous tissue that eventually contracts around the blood vessels, raising

pressure in the entire portal vascular system of the gastrointestinal tract;

this increases capillary hydrostatic pressure. This high capillary pressure

causes fluid to transude into the abdominal cavity, creating ascites. The

formation of ascitic fluid is further increased by lower than normal levels

of albumin (decreased plasma colloid osmotic pressure) because the

production of albumin is lower than normal in the diseased liver.

TMP14 pp. 317–318, 872

11. B) Assimilation of proteins means digestion and absorption of proteins.

Protein digestion begins in the stomach with the actions of pepsin (item 2).

Next, the presence of fat and protein in the small intestine stimulates the

release of cholecystokinin (item 3). Cholecystokinin then causes the release

of proteolytic enzymes from the pancreas (item 4). The protein digestion

products are then absorbed by enterocytes of the gut wall (item 1).

TMP14 pp. 824–825

12. E) Once the vomiting center has been sufficiently stimulated and the

vomiting act has been initiated, the first effects are (1) a deep breath, (2)

raising of the hyoid bone and larynx to pull the upper esophageal

sphincter open, (3) closing of the glois to prevent vomitus flow into the

lungs, and (4) lifting of the soft palate to close the posterior nares. Next

comes a strong downward contraction of the diaphragm along with

simultaneous contraction of all the abdominal wall muscles, which

squeezes the stomach between the diaphragm and the abdominal muscles,

building the intragastric pressure to a high level. Finally, the lower

esophageal sphincter relaxes completely, allowing expulsion of the gastric

contents upward through the esophagus and into the mouth.

TMP14 pp. 837–838

13. E) The electrochemical gradient for sodium results from active transport

of sodium ions through the basolateral membranes of the enterocytes and

into the adjacent interstitial spaces. This electrochemical gradient for

sodium powers the secondary active transport of many different nutrients

into the epithelial cells of the gut wall. These nutrients include glucose,

galactose, and several different amino acids.

TMP14 pp. 828–829

14. A) Arterial flow into the villus and the venous flow out of the villus are in

directions opposite to each other, and the vessels lie in close apposition to

each other. Because of this vascular arrangement, much of the blood

oxygen diffuses out of the arterioles directly into the adjacent venules

without ever being carried in the blood to the tips of the villi. As much as

80% of the oxygen may take this short-circuit route and is therefore not

available for local metabolic functions of the villi.

TMP14 pp. 795–796

15. D) When feces enter the rectum, distention of the rectal wall initiates

afferent signals that spread through the myenteric plexus to initiate

peristaltic waves in the descending colon, sigmoid, and rectum, forcing

feces toward the anus. As the peristaltic wave approaches the anus, the

internal anal sphincter is relaxed by inhibitory signals from the myenteric

plexus; if the external anal sphincter is also consciously, voluntarily relaxed

at the same time, defecation occurs.

TMP14 pp. 805–806

16. G) When food enters the stomach, the pH of the gastric contents increases

because the food buffers the acid (item 2). This increase in pH lowers

somatostatin secretion from delta cells in the stomach wall, which leads to

an increase in the rate of acid secretion (item 3). The pH of the gastric

contents remains high until the buffering capacity of the food is saturated.

Next, the pH of the gastric contents decreases (item 1), which stimulates

the secretion of somatostatin. Somatostatin decreases the rate of acid

secretion (item 4) by a direct action on parietal cells as well as by decreased

gastrin release from G-cells.

TMP14 p. 814

17. B) Gastrin is the only gastrointestinal hormone listed that is produced and

stored in the antrum of the stomach. All five gastrointestinal hormones are

produced and stored in the duodenum and jejunum.

TMP14 p. 792

18. E) During the swallowing process, the soft palate is pulled upward to

close the posterior nares to prevent reflux of food into the nasal cavities

(item 3). Next, the palatopharyngeal folds on each side of the pharynx are

pulled medially to approximate each other (item 4). The vocal cords of the

larynx are strongly approximated, and the larynx is pulled upward and

anteriorly by the neck muscles. The upper esophageal sphincter relaxes

(item 1), and peristaltic contractions of the pharynx (item 2) move the

bolus of food into the esophagus.

TMP14 pp. 797–798

19. B) The three secretagogues for gastric acid secretion are gastrin,

histamine, and acetylcholine. These secretagogues have a synergistic or

multiplicative action on acid secretion, which means that blocking the

action of one secretagogue will cause the other secretagogues to be less

effective. Hence, blocking the actions of histamine with an H2

 antagonist

causes gastrin and acetylcholine to be less effective in stimulating acid

secretion.

TMP14 p. 812

20. E) Helicobacter pylori is endemic in many underprivileged areas of the

world. At least 75% of persons with peptic ulcers have been found to have

chronic infection of the terminal portions of the gastric mucosa and initial

portions of the duodenal mucosa, most often caused by the bacterium H.

pylori. Once this infection begins, it can last a lifetime unless it is eradicated

by antibacterial therapy.

TMP14 p. 835

21. B) Patients with a lactase deficiency cannot digest milk products that

contain lactose (milk sugar). The operons of gut bacteria quickly switch

over to lactose metabolism, which results in fermentation that produces

copious amounts of gas (a mixture of hydrogen, carbon dioxide, and

methane). This gas, in turn, may cause a range of abdominal symptoms

including stomach cramps, bloating, and flatulence. The gas is absorbed by

blood (especially in the colon) and exhaled from the lungs. Blood glucose

levels do not increase because lactose is not digested to glucose and

galactose in these patients.

TMP14 pp. 823, 838

22. A) The appearance of mass movements after meals is facilitated by

gastrocolic and duodenocolic reflexes. These reflexes result from distention

of the stomach and duodenum. They are greatly suppressed when the

extrinsic autonomic nerves to the colon have been removed; therefore, the

reflexes are likely transmied by way of the autonomic nervous system.

All the gut reflexes are named with the anatomical origin of the reflex as

the prefix followed by the name of the gut segment in which the outcome

of the reflex is observed. For example, the duodenocolic reflex begins in

the duodenum and ends in the colon. When the duodenum is distended,

nervous signals are transmied to the colon, which stimulates mass

movements. The enterogastric reflex occurs when signals originating in the

intestines inhibit gastric motility and gastric secretion. The

intestinointestinal reflex occurs when overdistention or injury to a bowel

segment signals the bowel to relax. The rectosphincteric reflex, also called

the defecation reflex, is initiated when feces enter the rectum and stimulates

the urge to defecate.

TMP14 pp. 805–806

23. D) About 94% of the bile salts are reabsorbed into the blood from the

small intestine, with about half of this by diffusion through the mucosa in

the early portions of the small intestine and the remainder by an active

transport process through the intestinal mucosa in the distal ileum.

TMP14 p. 819

24. E) Vitamin B12 is absorbed in the ileum; this absorption requires intrinsic

factor, which is a glycoprotein secreted by parietal cells in the stomach.

Binding of intrinsic factor to dietary vitamin B12 is necessary for

aachment to specific receptors located in the brush border of the ileum.

Atrophic gastritis is a type of autoimmune gastritis that is mainly confined

to the acid-secreting corpus mucosa. The gastritis is diffuse, and severe

atrophy eventually develops. Ileal resection is likely to cause diarrhea but

not constipation. A gastric ulcer is possible but relatively unlikely. GERD is

caused by gastric acid and bile reflux into the esophagus; mucosal damage

and epithelial cell transformation lead to Barre esophagus, which is a

forerunner to adenocarcinoma, a particularly lethal cancer.

TMP14 p. 834

25. B) GIP is the only gastrointestinal hormone released by all three major

foodstuffs (fats, proteins, and carbohydrates). The presence of fat and

protein in the small intestine stimulates the release of CCK, but

carbohydrates do not stimulate its release. The presence of protein in the

antrum of the stomach stimulates the release of gastrin, but fat and

carbohydrates do not stimulate its release. Fat has a minor effect to

stimulate the release of motilin and secretin, but neither hormone is

released by the presence of protein or carbohydrate in the gastrointestinal

tract.

TMP14 p. 792

26. D) GIP is released by the presence of fat, carbohydrate, or protein in the

gastrointestinal tract. GIP is a strong stimulator of insulin release and is

responsible for the observation that an oral glucose load releases more

insulin and is metabolized more rapidly than an equal amount of glucose

administered intravenously. Intravenously administered glucose does not

stimulate the release of GIP. Neither CCK nor VIP stimulates the release of

insulin. GIP does not stimulate glucagon release, and glucagon has the

opposite effect of insulin; that is, it would decrease the rate of glucose

clearance from the blood. VIP does not stimulate GIP release.

TMP14 p. 792

27. C) Achlorhydria means simply that the stomach fails to secrete

hydrochloric acid. This condition is diagnosed when the pH of the gastric

secretions fails to decrease below 4 after stimulation by pentagastrin.

When acid is not secreted, pepsin also usually is not secreted. Even when it

is, the lack of acid prevents it from functioning because pepsin requires an

acid medium for activity. Thus, protein digestion is impaired.

TMP14 pp. 812, 834

28. C) Achalasia is a condition in which the LES fails to relax during

swallowing. As a result, food swallowed into the esophagus fails to pass

from the esophagus into the stomach. Trace C shows a high, positive

pressure that fails to decrease after swallowing, which is indicative of

achalasia. Trace A shows a normal pressure tracing at the level of the LES,

reflecting typical receptive relaxation in response to the food bolus. Trace E

is similar to trace C, but the pressures are subatmospheric. Subatmospheric

pressures occur only in the esophagus where it passes through the chest

cavity.

TMP14 pp. 799, 833

29. D) Pepsinogen is the precursor of the enzyme pepsin. Pepsinogen is

secreted from the peptic or chief cells of the gastric gland (also called the

oxyntic gland). To be converted from the precursor form to the active form

(pepsin), pepsinogen must come in contact with hydrochloric acid or

pepsin itself. Pepsin is a proteolytic enzyme that digests collagen and other

types of connective tissue in meats.

TMP14 pp. 811–812

30. A) Cholecystokinin (CCK) is the only gastrointestinal hormone that

inhibits gastric emptying under physiological conditions. This inhibition of

gastric emptying keeps the stomach full for a prolonged time, which is one

reason why a breakfast containing fat and protein “sticks with you” beer

than breakfast meals containing mostly carbohydrates. CCK also has a

direct effect on the feeding centers of the brain to reduce further eating.

Although CCK is the only gastrointestinal hormone that inhibits gastric

emptying, all the gastrointestinal hormones except for gastrin are released

to some extent by the presence of fat in the intestine.

TMP14 p. 792

31. F) All these factors can inhibit gastric acid secretion under normal

physiological conditions. Gastric acid stimulates the release of

somatostatin (a paracrine factor), which has a direct effect on the parietal

cell to inhibit acid secretion, as well as an indirect effect mediated by

suppression of gastrin secretion. Secretin and GIP inhibit acid secretion

through a direct action on parietal cells as well as indirectly through

suppression of gastrin secretion. Enterogastrones are unidentified

substances released from the duodenum and jejunum that directly inhibit

acid secretion. When acid or hypertonic solutions enter the duodenum, a

neurally mediated decrease in gastric acid secretion follows.

TMP14 p. 814

32. B) Gastrin stimulates gastric acid secretion, and secretin and GIP inhibit

gastric acid secretion under normal physiological conditions. It is

important to differentiate the physiological effects of the gastrointestinal

hormones from their pharmacologic actions. For example, gastrin and

CCK have identical actions on gastrointestinal function when large

pharmacologic doses are administered, but they do not share any actions

at normal physiological concentrations. Likewise, GIP and secretin share

multiple actions when pharmacologic doses are administered, but only one

action is shared at physiological concentrations: inhibition of gastric acid

secretion.

TMP14 p. 792

33. E) The cephalic phase of gastric secretion occurs before food enters the

stomach. Seeing, smelling, chewing, and anticipating food are perceived

by the brain, which “tells” the stomach to prepare for a meal. Stimuli for

the cephalic phase thus include mechanoreceptors in the mouth,

chemoreceptors (smell and taste), thought of food, and hypoglycemia.

Because the cephalic phase of gastric secretion is mediated entirely by way

of the vagus nerves, vagotomy can abolish the response. Antacids

neutralize gastric acid, but they do not inhibit gastric secretion. An

antigastrin antibody would aenuate (but not abolish) the cephalic phase

because this would have no direct effect on histamine and acetylcholine

stimulation of acid secretion. Atropine would aenuate the cephalic phase

by blocking acetylcholine receptors on parietal cells; however, atropine

does not abolish acetylcholine stimulation of gastrin secretion. A histamine

H2

 blocker would aenuate the cephalic phase of gastric secretion but

would not abolish it.

TMP14 p. 813

34. C) MMCs (sometimes called interdigestive myoelectric complexes) are

peristaltic waves of contraction that begin in the stomach and slowly

migrate in an aboral direction along the entire small intestine to the colon.

By sweeping undigested food residue from the stomach, through the small

intestine, and into the colon, MMCs function to maintain low bacterial

counts in the upper intestine. Bacterial overgrowth syndrome can occur

when the normally low bacterial colonization in the upper gastrointestinal

tract increases significantly. It should be clear that an absence of MMCs

would decrease duodenal motility and gastric emptying. MMCs do not

have a direct effect on mass movements and swallowing.

TMP14 pp. 792–793

35. C) Trace C shows a basal subatmospheric pressure with a positive

pressure wave caused by passage of the food bolus. Trace A does not

correspond to any normal event in the esophagus. Trace B could represent

the LES in a patient with achalasia. Trace D depicts normal operation of

the LES. Trace E shows a basal positive pressure trace, which does not

occur where the esophagus passes through the chest cavity.

TMP14 p. 799

36. C) Gastric emptying is accomplished by coordinated activities of the

stomach, pylorus, and small intestine. Conditions that favor gastric

emptying include (a) increased tone of the orad stomach, which helps to

push chyme toward the pylorus; (b) forceful peristaltic contractions in the

stomach that move chyme toward the pylorus; (c) relaxation of the

pylorus, which allows chyme to pass into the duodenum; and (d) absence

of segmentation contractions in the intestine, which can otherwise impede

the entry of chyme into the intestine.

TMP14 pp. 800–801

37. A) The toxin from V. cholerae (cholera toxin) causes an irreversible increase

in cAMP levels (not cGMP levels) in the enterocytes located in the crypts of

Lieberkühn of the small intestine. This increase in cAMP causes an

irreversible opening of chloride channels on the luminal membrane.

Movement of chloride ions into the gut lumen causes a secondary

movement of sodium ions to maintain electrical neutrality. Water follows

the osmotic gradient created by sodium and chloride, causing a

tremendous increase in fluid loss into the gut lumen. Severe diarrhea

follows.

TMP14 pp. 830, 836

38. B) Cholera toxin causes an irreversible opening of chloride channels in the

enterocytes located in the crypts of Lieberkühn of the small intestine, as

indicated in the explanation for the previous answer. Although sodium

ions enter the gut lumen to maintain electrical neutrality after the flux of

chloride ions into the gut lumen, the sodium ions move through relatively

large paracellular pathways rather than through actual sodium channels.

Calcium, potassium, and magnesium do not have a significant role during

an infection with V. cholerae.

TMP14 pp. 830, 836

39. B) Enterocytes are derived from stem cells located in the crypts of

Lieberkühn of the small intestine. They mature as they migrate upward

toward the villus tip, where they are extruded into the gut lumen,

becoming part of the ingesta. In humans, the entire population of epithelial

cells is replaced in 3 to 6 days. Cholera also usually runs its course in 3 to 6

days. Because cholera toxin causes an irreversible opening of chloride

channels in the enterocytes, it is thought that the time course of cholera is

dictated by the life span of the enterocytes.

TMP14 pp. 830, 836

40. E) Cholecystokinin (CCK) is the only gastrointestinal hormone that

inhibits gastric emptying under normal physiological conditions. CCK

inhibits gastric emptying by relaxing the orad stomach, which increases its

compliance. When the compliance of the stomach is increased, the stomach

can hold a larger volume of food without excess buildup of pressure in the

lumen. None of the gastrointestinal hormones increases gastric emptying

under physiological conditions; however, gastrin, secretin, and GIP can

inhibit gastric emptying when pharmacologic doses are administered

experimentally.

TMP14 p. 792

41. E) The figure shows the time course of gastric pH, rate of acid secretion,

and stomach volume immediately before and for 4 hours after a meal.

Note that the pH of the gastric juice is lowest immediately before the meal

(not an answer choice) and 4 hours after consuming the meal (the correct

answer). It is a common misconception that the pH of the gastric juice is

lowest (most acidic) after a meal, when acid secretion is highest.

TMP14 pp. 813–814

42. A) Gastrin and CCK do not share any effects on gastrointestinal function

at normal physiological conditions; however, they have identical actions

on gastrointestinal function when pharmacologic doses are administered.

Gastrin stimulates gastric acid secretion and mucosal growth throughout

the stomach and intestines under physiological conditions. CCK stimulates

growth of the exocrine pancreas and inhibits gastric emptying under

normal conditions. CCK also stimulates gallbladder contraction, relaxation

of the sphincter of Oddi, and secretion of bicarbonate and enzymes from

the exocrine pancreas.

TMP14 p. 792

43. A) The frequency of slow waves is fixed in various parts of the gut. The

maximum frequency of smooth muscle contractions cannot exceed the

slow-wave frequency. The slow-wave frequency averages about 3 per

minute in the stomach, 12 per minute in the duodenum, 10 per minute in

the jejunum, and 8 per minute in the ileum. Therefore, the duodenum is

most likely to have the highest frequency of smooth muscle contractions.

TMP14 pp. 787–788

44. D) When feces enter the rectum, distention of the rectal wall initiates

signals that spread through the myenteric plexus to initiate peristaltic

waves in the descending colon, sigmoid colon, and rectum, all of which

force feces toward the anus. At the same time, the internal anal sphincter

relaxes, allowing the feces to pass. In people with transected spinal cords,

the defecation reflexes can cause automatic emptying of the bowel because

the external anal sphincter is normally controlled by the conscious brain

through signals transmied in the spinal cord.

TMP14 pp. 805–806

45. B) The use of NSAIDs may result in NSAID-associated gastritis or peptic

ulceration. Chronic gastritis, by definition, is a histopathologic entity

characterized by chronic inflammation of the stomach mucosa. When

inflammation affects the gastric corpus, parietal cells are inhibited, leading

to reduced acid secretion. Although diagnosis of chronic gastritis can only

be ascertained histologically, the administration of pentagastrin should

produce a less than expected increase in gastric acid secretion. Pentagastrin

is a synthetic gastrin composed of the terminal four amino acids of natural

gastrin plus the amino acid alanine. It has all the same physiological

properties of natural gastrin. Although gastrin and pentagastrin can both

stimulate growth of the duodenal mucosa, it should be clear that

intravenous pentagastrin would not cause substantial growth in the

context of a clinical test. In any case, chronic administration of pentagastrin

would not lead to a less than expected growth of the duodenal mucosa.

Pentagastrin is not expected to increase gastrin secretion, pancreatic

enzyme secretion, or pancreatic growth.

TMP14 pp. 833–834

46. E) The presence of acid, fay acids, and hyperosmotic solutions in the

duodenum and jejunum leads to suppression of acid secretion through a

variety of mechanisms. Acid stimulates the secretion of secretin from the

small intestine, which in turn inhibits acid secretion from parietal cells.

Acidification of the antrum and oxyntic gland area of the stomach

stimulates the release of somatostatin, which in turn inhibits acid secretion

by a direct action on the parietal cells and an indirect action mediated by

suppression of gastrin secretion. The presence of fay acids in the small

intestine stimulates the release of GIP, which inhibits acid secretion both

directly (parietal cell inhibition) and indirectly (by decreasing gastrin

secretion). Hyperosmotic solutions in the small intestine cause the release

of unidentified enterogastrones, which directly inhibit acid secretion from

parietal cells. Isotonic solutions have no effect on acid secretion.

TMP14 p. 814

47. C) Before a meal, when the stomach is empty, the pH of the gastric juice is

at its lowest point, and acid secretion is suppressed. Acid secretion is

suppressed in part because (a) the concentrated hydrogen ions in the

gastric juice stimulate somatostatin release, which has a direct action to

decrease the secretion of both gastrin and acid, and (b) the acid itself has a

direct effect to suppress parietal cell secretions. When a meal is taken, the

buffering effects of the food cause the gastric pH to increase, which in turn

decreases somatostatin release. Cholecystokinin and vasoactive intestinal

peptide do not have a role in the regulation of gastric acid secretion.

TMP14 p. 814

48. D) The act of vomiting is preceded by antiperistalsis that may begin as far

down in the gastrointestinal tract as the ileum. Distention of the upper

portions of the gastrointestinal tract (especially the duodenum) becomes

the exciting factor that initiates the actual act of vomiting. At the onset of

vomiting, strong contractions occur in the duodenum and stomach along

with partial relaxation of the lower esophageal sphincter. From then on, a

specific vomiting act ensues that involves (a) a deep breath, (b) relaxation

of the upper esophageal sphincter, (c) closure of the glois, and (d) strong

contractions of the abdominal muscles and diaphragm.

TMP14 pp. 837–838

49. E) Secretin inhibits gastrin secretion from normal G-cells in the antrum

and duodenum but stimulates gastrin secretion in gastrinoma cells. Any

increase in serum gastrin concentration greater than 110 pg/ml above

baseline after administration of human secretin is diagnostic of gastrinoma

(also called Zollinger-Ellison syndrome). The secretin test is considered the

most sensitive and accurate diagnostic method for gastrinoma.

TMP14 pp. 792, 814

50. E) Essentially all proteolytic enzymes are secreted in an inactive form,

which prevents autodigestion of the secreting organ. Enterokinase is

physically aached to the brush border of the enterocytes that line the

inner surface of the small intestine. Enterokinase activates trypsinogen to

become trypsin in the gut lumen. The trypsin then catalyzes the formation

of additional trypsin from trypsinogen, as well as several other

proenzymes (e.g., chymotrypsinogen, procarboxypeptidase, proelastase).

Pepsin is first secreted as pepsinogen, which has no proteolytic activity.

However, as soon as it comes into contact with hydrochloric acid, and

especially in contact with previously formed pepsin plus hydrochloride

acid, it is activated to form pepsin.

TMP14 p. 815

51. B) Neither plasma gastrin levels nor the rate of acid secretion is diagnostic

for duodenal ulcer. However, when patients with a duodenal ulcer are

pooled together, they exhibit a statistically significant increase in the rate

of acid secretion and a statistically significant decrease in plasma gastrin

levels. How is this possible? The basal and maximal acid secretion rates of

normal subjects range from 1 to 5 mEq/h and from 6 to 40 mEq/h,

respectively, which overlaps with the basal (2-10 mEq/h) and maximal (30-

80 mEq/h) acid secretion rates of persons with a duodenal ulcer. The

increase in acid secretion of the average person with a duodenal ulcer

suppresses the secretion of gastrin from the antrum of the stomach. It

should be obvious that endoscopy is diagnostic for duodenal ulcer.

TMP14 pp. 834–835

52. E) All of the gastrointestinal hormones are released after a meal, and all

have physiological effects.

TMP14 p. 792

53. A) H. pylori is a bacterium that accounts for 95% of patients with a

duodenal ulcer and virtually 100% of patients with a gastric ulcer when

chronic use of aspirin or other NSAIDs are eliminated. H. pylori is

characterized by high urease activity, which metabolizes urea to NH3

(ammonia). Ammonia reacts with H+

 to become ammonium (NH4

+

). This

reaction allows the bacterium to withstand the acid environment of the

stomach. The ammonium production is believed to be the major cause of

cytotoxicity because the ammonium directly damages epithelial cells,

increasing the permeability of the gastric mucosal barrier. Bile salts and

NSAIDs can also damage the gastric mucosal barrier, but these substances

are not directly related to H. pylori infection. Pepsin can exacerbate the

mucosal lesions caused by H. pylori infection, but pepsin levels are not

increased by H. pylori. It should be clear that gastrin does not mediate the

mucosal damage caused by H. pylori.

TMP14 p. 835

54. B) Slow-wave frequency is not affected significantly by either the

autonomic nervous system or hormones; it is relatively constant at any

given location in the small intestine. When a slow wave reaches a

threshold value, a calcium spike potential (action potential) occurs, and

calcium ions enter the smooth muscle cell, which causes it to contract.

Norepinephrine hyperpolarizes smooth muscle cells in the intestine and

thereby decreases the likelihood that the membrane potential can reach a

threshold value. Therefore, norepinephrine does not affect the basal slowwave frequency of 10 occurrences per minute but does lower the

contraction frequency of the smooth muscle cells to zero occurrences per

minute in this problem.

TMP14 pp. 788, 802–803

55. E) The defecation reflex (also called the rectosphincteric reflex) occurs when

feces enter the rectum. When the rectum is stretched, the internal anal

sphincter relaxes, and the rectum contracts, pushing the feces toward the

anus. The external anal sphincter is controlled voluntarily and can be

contracted when defecation is not possible. Therefore, when a person feels

the urge to defecate, the internal anal sphincters are relaxed, the rectum

contracts, and the external anal sphincter is either contracted or relaxed

depending on the circumstances.

TMP14 pp. 805–806

56. E) Oxygen is shunted from the artery of a villus into its venous drainage

so that by the time the arterial blood reaches the villus tip, the oxygen

tension has been reduced to about 10 mm Hg. Adenosine dilates the villus

artery, increasing blood flow to the villus tip. This increase in blood flow

decreases the residence time for blood in the artery so that greater amounts

of oxygen can reach the villus tip, thus increasing the oxygen tension at the

villus tip. Factors that decrease intestinal blood flow (e.g., hemorrhagic

shock and a severe degree of exercise) can lead to ischemic death of villi

because of their low level of oxygenation.

TMP14 pp. 795–796

57. E) One of the most critical actions of gastrointestinal hormones is their

trophic activity. Gastrin can stimulate mucosal growth throughout the

gastrointestinal tract as well as growth of the exocrine pancreas. If most of

the endogenous gastrin is removed by antrectomy, the gastrointestinal

tract atrophies. Exogenous gastrin prevents the atrophy. Partial resection

of the small intestine for tumor removal, morbid obesity, or other reasons

results in hypertrophy of the remaining mucosa. The mechanism for this

adaptive response is poorly understood. Both cholecystokinin and secretin

stimulate growth of the exocrine pancreas. GIP and motilin do not appear

to have trophic actions on the gastrointestinal tract.

TMP14 p. 792

58. B) Relaxation of the ileocecal sphincter occurs with or shortly after eating.

This reflex has been termed the gastroileal reflex. It is not clear whether the

reflex is mediated by gastrointestinal hormones (gastrin and

cholecystokinin) or extrinsic autonomic nerves to the intestine. Note that

the gastroileal reflex is named with the origin of the reflex first (gastro) and

the target of the reflex named second (ileal). This method of naming is

characteristic of all the gastrointestinal reflexes. The enterogastric reflex

involves signals from the colon and small intestine that inhibit gastric

motility and gastric secretion. The gastrocolic reflex causes the colon to

evacuate when the stomach is stretched. The intestino-intestinal reflex

causes a bowel segment to relax when it is overstretched. The

rectosphincteric reflex is also called the defecation reflex.

TMP14 p. 803

59. D) Damage to the gastric mucosal barrier allows hydrogen ions to backleak into the mucosa in exchange for sodium ions. A low pH in the mucosa

causes mast cells to leak histamine, which damages the vasculature,

causing ischemia. The ischemic mucosa allows a greater leakage of

hydrogen ions—leading to more cell injury and death—resulting in a

vicious cycle. Factors that normally strengthen the gastric mucosal barrier

include mucus (which impedes the influx of hydrogen ions), gastrin

(which stimulates mucosal growth), certain prostaglandins (which can

stimulate mucus secretion), and various growth factors that can stimulate

growth of blood vessels, gastric mucosa, and other tissues. Factors that

weaken the gastric mucosal barrier include H. pylori (a bacterium that

produces toxic levels of ammonium), as well as aspirin, NSAIDs, ethanol,

and bile salts.

TMP14 pp. 833–834

60. B) Fat entering the small intestine is first emulsified into smaller globules

by bile released from the gallbladder. Pancreatic lipase in conjunction with

the co-enzyme colipase then digests the fat (which is mostly triglycerides)

into monoglycerides and free fay acids; these substances then become

surrounded by bile salts to form water-soluble aggregates called micelles.

When a micelle makes contact with an enterocyte of the intestinal wall, the

monoglycerides and free fay acids diffuse directly through the cell

membrane into the enterocyte; triglycerides are too large to be absorbed.

Once inside the enterocyte, the monoglycerides and free fay acids form

new triglyceride molecules that are subsequently packaged by the Golgi

apparatus into chylomicrons. The chylomicrons exocytose at the

basolateral membrane of the enterocyte and enter a lymphatic capillary

(central lacteal) in the villus.

TMP14 p. 826

61. C) The medical treatment of gastric ulcers is aimed at restoring the

balance between acid secretion and mucosal protective factors. Proton

pump inhibitors are drugs that covalently bind and irreversibly inhibit the

H+

/K+

 adenosine triphosphatase (ATPase) pump, effectively inhibiting acid

release. Therapy can also be directed toward histamine release, that is, H2

blockers, such as cimetidine (Tagamet), ranitidine (Zantac), famotidine

(Pepcid), and nizatidine (Axid). These agents selectively block the H2

receptors in the parietal cells. Antibiotic therapy is used to eradicate the H.

pylori infection. NSAIDs can cause damage to the gastric mucosal barrier,

which is a forerunner of gastric ulcer.

TMP14 p. 835

62. B) Movement of chloride ions out of cells leads to secretion of fluid by

cells. CF is caused by abnormal chloride ion transport on the apical surface

of epithelial cells in exocrine gland tissues. The CF transmembrane

regulator (CFTR) protein functions both as a cAMP-regulated Cl − channel

and, as its name implies, a regulator of other ion channels. The fully

processed form of CFTR is found in the plasma membrane of normal

epithelia. Absence of CFTR at appropriate cellular sites is often part of the

pathophysiology of CF. However, other mutations in the CF gene produce

CFTR proteins that are fully processed but are nonfunctional or only

partially functional at the appropriate cellular sites.

TMP14 p. 830

63. A) Hydrogen ions leak into the mucosa when it is damaged. As the

hydrogen ions accumulate in the mucosa, the intracellular buffers become

saturated, and the pH of the cells decreases, resulting in injury and cell

death. The hydrogen ions also damage mast cells, causing them to secrete

excess amounts of histamine. The histamine exacerbates the condition by

damaging blood capillaries within the mucosa. The result is focal ischemia,

hypoxia, and vascular stasis. The mucosal lesion is a forerunner of gastric

ulcer. Mucus secretion helps strengthen the gastric mucosal barrier

because mucus impedes the leakage of hydrogen ions into the mucosa.

Various proton pump inhibitors are used as a treatment modality for

gastric ulcers because these inhibitors can decrease the secretion of

hydrogen ions (protons). The tight junctions between cells within the

mucosa help prevent the back-leak of hydrogen ions. Vagotomy was once

used to treat gastric ulcer disease because severing or crushing the vagus

nerve decreases gastric acid secretion.

TMP14 pp. 833–834

64. D) The fundus of the stomach and lower esophageal sphincter both relax

during a swallow while the bolus of food is still higher in the esophagus.

This phenomenon is called receptive relaxation. Receptive relaxation is

mediated by afferent and efferent pathways in the vagus nerves. Nitric

oxide is the neurotransmier thought to mediate receptive relaxation at the

smooth muscle cell. Motilin is a gastrointestinal hormone that mediates

migrating motility complexes (also called housekeeping contractions); these

contractions occur between meals. Gastrin and histamine do not have

significant effects on smooth muscle contraction or relaxation at

physiological levels. Norepinephrine can decrease smooth muscle

contraction in the small intestine but is not involved in receptive

relaxation.

TMP14 p. 799

65. A) Mass movements force feces into the rectum. When the walls of the

rectum are stretched by the feces, the defecation reflex is initiated, and a

bowel movement follows when this is convenient. Mass movements do not

affect gastric motility. Haustrations are bulges in the large intestine caused

by contraction of adjacent circular and longitudinal smooth muscle. It

should be clear that mass movements in the colon do not affect esophageal

contractions or pharyngeal peristalsis.

TMP14 pp. 805–806

66. E) This woman has gastroparesis (also called delayed gastric emptying). This

disorder slows or at times even stops the movement of chyme from the

stomach to the duodenum. Diabetes is the most commonly known cause of

gastroparesis; it occurs in about 20% of persons with type 1 diabetes. The

high blood glucose level is thought to damage the vagus nerves and

thereby delay gastric emptying.

TMP14 pp. 800–801

67. C) Acid acts directly on somatostatin cells to stimulate the release of

somatostatin. The somatostatin decreases acid secretion by directly

inhibiting the acid-secreting parietal cells and indirectly by inhibiting

gastrin secretion from G-cells in the antrum. Acid is a weak stimulus for

CCK release, but CCK does not inhibit (or stimulate) gastrin release. Acid

does not stimulate GIP release. Fay acids are a weak stimulus for motilin,

but motilin does not affect gastrin release. Fay acids are not thought to

stimulate somatostatin release.

TMP14 pp. 792, 814

68. C) All five gastrointestinal hormones are released from both the

duodenum and jejunum. Only gastrin is released from the antrum. Small

amounts of cholecystokinin and secretin are also released from the ileum.

No gastrointestinal hormones are released from the colon or esophagus.

TMP14 p. 792

69. D) Primary peristalsis of the esophagus is a continuation of pharyngeal

peristalsis; central control originates in the swallowing center located in

the medulla and pons. Visceral somatic fibers in the vagus nerves directly

innervate smooth muscle fibers of the pharynx and upper esophagus,

which coordinate pharyngeal peristalsis and primary peristalsis of the

esophagus. Esophageal contractions can occur independently of vagal

stimulation by a local stretch reflex initiated by the food bolus itself; this

phenomenon is called secondary peristalsis. Although the vagus nerves can

stimulate gastric acid secretion, gastrin release, and pancreatic bicarbonate

secretion, these processes can be activated by other mechanisms. Thus,

elimination of vagal stimulation does not completely eliminate them.

TMP14 p. 799

70. D) Persons with duodenal ulcers have about 2 billion parietal cells and

can secrete about 40 mEq H+

 per hour. Unaffected individuals have about

50% of these values. Plasma gastrin levels are related inversely to acid

secretory capacity because of a feedback mechanism by which antral

acidification inhibits gastrin release. Thus, plasma gastrin levels are

usually reduced in persons with duodenal ulcers. Maximal acid secretion

and plasma gastrin levels are not diagnostic for duodenal ulcer disease

because of significant overlap with the normal population among persons

in each group.

TMP14 pp. 834–835

71. A) Intrinsic factor is a glycoprotein secreted from parietal cells (i.e., acidsecreting cells in the stomach) that is necessary for absorption of vitamin

B12. The patient has a diminished capacity to secrete acid because of

chronic gastritis. Because acid and intrinsic factor are both secreted by

parietal cells, a diminished capacity to secrete acid is usually associated

with diminished capacity to secrete intrinsic factor. Ptyalin, also known as

salivary amylase, is an enzyme that begins carbohydrate digestion in the

mouth. The secretion of ptyalin is not affected by gastritis. Rennin, known

also as chymosin, is a proteolytic enzyme synthesized by chief cells in the

stomach. Its role in digestion is to curdle or coagulate milk in the stomach,

a process of considerable importance in very young animals. It should be

clear that saliva secretion is not affected by gastritis. Trypsin is a

proteolytic enzyme secreted by the pancreas.

TMP14 p. 834

72. C) Parietal cells have receptors for all four substances shown. Stimulation

of receptors for gastrin, acetylcholine, and histamine lead to increased

secretion of gastric acid; stimulation of somatostatin receptors inhibits

gastric acid secretion.

TMP14 p. 813

73. C) This woman has celiac disease, also called gluten-sensitive

enteropathy, which is a chronic disease of the digestive tract that interferes

with the absorption of nutrients from food. Mucosal lesions seen on upper

gastrointestinal biopsy specimens are the result of an abnormal, genetically

determined, cell-mediated immune response to gliadin, a constituent of

the gluten found in wheat; a similar response occurs to comparable

proteins found in rye and barley. Gluten is not found in oats, rice, or corn.

When persons with celiac disease ingest gluten, the mucosa of their small

intestine is damaged by an immunologically mediated inflammatory

response, which results in malabsorption and maldigestion at the brush

border. Digestion of fat is normal in persons with celiac disease because

lipase secreted by the pancreas still functions normally. Malabsorption in

celiac disease increases the stool content of carbohydrates, fat, and

nitrogen. There is no cure for celiac disease, but a strict gluten-free diet can

help manage symptoms and promote intestinal healing.

TMP14 pp. 835–836

74. B) This infant has Hirschsprung’s disease, which is characterized by a

congenital absence of ganglion cells in the distal colon, resulting in a

functional obstruction. Prolonged fecal stasis can lead to enterocolitis (i.e.,

inflammation of the colon); full-thickness necrosis and perforation can

occur in severe cases. In achalasia, the lower esophageal sphincter fails to

relax during swallowing. Halitosis (bad breath) can occur in persons with

Hirschsprung’s disease, but this condition is not serious. Peptic ulcer and

pancreatitis (inflammation of the pancreas) are not common in persons

with Hirschsprung’s disease.

TMP14 p. 836

75. C) About 20% of persons older than 65 years have gallstones

(cholelithiasis) in the United States, and 1 million newly diagnosed cases of

gallstones are reported each year. Gallstones are the most common cause

of biliary obstruction. Regardless of the cause of gallstones, serum

bilirubin values (especially direct or conjugated) are usually elevated.

Indirect or unconjugated bilirubin values are usually normal or only

slightly elevated. Only choice C shows a high level of direct bili rubin

(conjugated bilirubin) compared with the level of indirect bilirubin

(unconjugated bilirubin).

TMP14 pp. 835, 874–875

76. A) Intestinal absorption of immunoglobulins (present in colostrum)

during early infancy occurs by endocytosis in the duodenum and jejunum.

This ability to absorb large molecules by endocytosis occurs during the

first several months of life but does not occur thereafter (except in the

ileum for absorption of vitamin B12). Facilitated diffusion, passive

diffusion, and primary and secondary active transport are all normal

transport processes in enterocytes.

TMP14 pp. 830–831

77. C) Pancreatitis is inflammation of the pancreas. The pancreas secretes

digestive enzymes into the small intestine that are essential in the digestion

of fats, proteins, and carbohydrates. Reduced secretion of fluid into the

pancreatic ducts in cystic fibrosis (CF) cause these digestive enzymes to

accumulate in the ducts. The digestive enzymes then become activated in

the pancreatic ducts (which typically would not occur) and can begin to

“digest” the pancreas, leading to inflammation and a myriad of other

problems (cysts and internal bleeding). Enterokinase is located at the brush

border of intestinal enterocytes, where it normally activates trypsin from

its precursor, trypsinogen. Trypsin inhibitor is normally present in the

pancreatic ducts, where it prevents trypsin from being activated and thus

prevents autodigestion of the pancreas. When the ducts are blocked in

cystic fibrosis, the available trypsin inhibitor is insufficient to prevent

trypsin from being activated. Excessive secretion of CCK does not occur in

persons with CF. Gallstone obstruction can lead to pancreatitis (by

autodigestion) when the obstruction prevents pancreatic juice from

entering the intestine, but this is unrelated to CF.

TMP14 pp. 815, 835


Unit XIII: Metabolism and

Temperature Regulation

1. A 54-year-old man eats a bowl of ice cream. Fifty minutes later,

chylomicrons enter his venous system through the thoracic duct.

Which of the following best describes the major constituent of a

typical chylomicron in this man?

A) Apoprotein B

B) Cholesterol

C) Monoglycerides

D) Phospholipids

E) Triglycerides

2. All the following tissues can use fay acids for energy EXCEPT

one. Which one is this EXCEPTION?

A) Brain

B) Heart

C) Kidney

D) Liver

E) Skeletal muscle

3. A 56-year-old woman with chronic liver disease is brought to the

physician because of confusion, a depressed level of consciousness,

personality changes, and intellectual impairment. Laboratory tests

support a diagnosis of liver encephalopathy. Which of the

following blood factors is the most likely cause of this condition?

A) Ammonia

B) Arginine

C) Citrulline

D) Ornithine

E) Urea

4. Elimination of bilirubin from the body requires several steps under

normal conditions that include (1) conjugation of bilirubin with

glucuronic acid, (2) carriage of bilirubin by albumin in plasma, (3)

excretion of bilirubin into bile, and (4) uptake of free bilirubin by

hepatocytes. Which of the following best describes the correct

temporal order of events?

A) 4, 3, 2, 1

B) 3, 1, 4, 2

C) 3, 4, 1, 2

D) 2, 1, 4, 3

E) 4, 2, 1, 3

F) 2, 4, 1, 3

G) 1, 2, 3, 4

H) 1, 3, 2, 4

5. A 45-year-old physician binge drinks alcohol three to five times per

week and does not eat during the alcoholic binges. Which of the

following long-term complications are possible in this man?

Jaundice Ascites Esophageal Varices Peripheral Edema

A) No Yes Yes No

B) No No Yes Yes

C) Yes Yes No Yes

D) Yes Yes Yes No

E) Yes Yes Yes Yes

6. A 54-year-old woman visits the physician because of upper

abdominal pain and vomiting. Physical examination shows upper

abdominal tenderness and diminished bowel sounds. Blood tests

show a 3-fold increase in plasma amylase levels compared with

normal. CT scan shows a mass at the papilla of Vater. Activation of

which of the following substances is the most likely cause of this

woman’s condition?

A) Chymotrypsin

B) Enterokinase

C) Lipase

D) Trypsin inhibitor

E) Trypsinogen

7. A 64-year-old, unacclimatized man works outside on a hot day.

Compared with an acclimatized person, this unacclimatized man is

most likely to have which of the following?

A) Decreased loss of sodium chloride in sweat only

B) Decreased sweat production only

C) Decreased sweat production and increased loss of sodium

chloride in sweat

D) Increased loss of sodium chloride in sweat only

E) Increased sweat production only

F) Increased sweat production and decreased loss of sodium

chloride in sweat

8. A 29-year-old man sits nude in a room that has a temperature of

71°F and relative humidity of 50%. He is 6 feet, 2 inches tall and

weighs 205 lb (body mass index = 26.3). The greatest amount of

body heat is most likely to be lost by which of the following

mechanisms?

A) Conduction to air

B) Conduction to objects

C) Evaporation

D) Radiation

9. The hypothalamic set-point temperature normally averages about

98.6°F. Which of the following factors can alter the set-point level

for core temperature control?

A) Skin temperature only

B) Pyrogens only

C) Thyroxin only

D) Skin temperature and pyrogens only

E) Skin temperature, pyrogens, and thyroxin

10. Which of the following describes the neurotransmier released

from neurons that innervate sweat glands?

A) Norepinephrine

B) Acetylcholine

C) Epinephrine

D) Dopamine

E) Glycine

11. A 23-year-old student gets lost in a snowstorm and does not have

proper clothing or supplies to make a fire. The body temperature

of the student decreases slowly over the next 18 hours. At which of

the following temperatures is the ability of the hypothalamus to

regulate body temperature completely lost (in °F)?

A) 95.0

B) 92.5

C) 90.0

D) 87.5

E) 85.0

12. Fay acid degradation in mitochondria produces which twocarbon substance?

A) Acetyl coenzyme A

B) Carnitine

C) Glycerol

D) Glycerol 3-phosphate

E) Oxaloacetic acid

13. The following events occurred during the course of a fever in a 12-

year-old boy: (1) cutaneous vasodilation and sweating; (2) a return

of the set-point temperature to normal; (3) an increase in the setpoint temperature to 103°F; and (4) shivering, chills, and cutaneous

vasoconstriction. Which of the following best describes the correct

temporal order of events during the fever in this boy?

A) 4, 3, 2, 1

B) 3, 4, 2, 1

C) 2, 1, 4, 3

D) 4, 2, 1, 3

E) 3, 4, 1, 2

F) 1, 2, 3, 4

G) 2, 3, 1, 4

H) 1, 3, 2, 4

14. A 72-year-old man with a 25-year history of alcoholism and liver

disease visits his physician because of sudden weight gain. One

year ago, the man had a body mass index (BMI) of 24.9 kg/m2

;

today his BMI is 28.5 kg/m2

. Physical examination shows +3 edema

in his feet and moderate ascites. Which condition is most likely to

have promoted the development of both ascites and peripheral

edema in this man?

A) Decreased capillary hydrostatic pressure

B) Decreased plasma colloid osmotic pressure

C) Increased capillary hydrostatic pressure

D) Increased plasma colloid osmotic pressure

15. A 24-year-old student goes hiking in the Mojave Desert during

spring break. The environmental temperature is 105°F, and the

relative humidity is 20%. Which option best describes the major

mechanism of heat loss in this student?

A) Conduction to air

B) Conduction to objects

j

C) Convection

D) Evaporation

E) Radiation

16. A 32-year-old student consumes a meal containing 10% fat, 50%

carbohydrate, and 40% protein. Four hours later, the metabolic rate

has increased by about 30%, even though the student is siing at

rest. Which substance is most likely to cause the greatest increase

in metabolic rate in this student 4 hours after consuming the meal?

A) Carbohydrate

B) Fat

C) Protein

17. A 90-year-old man is found siing in his yard, vomiting on a hot

summer day with the lawnmower running. The man is confused

and dizzy. He is admied to the hospital as an emergency patient.

His body temperature is 105°F, his heart rate is 110 beats/min, and

his skin turgor is poor. Which symptom is unlikely in this man?

A) Headache

B) Hot skin

C) Hypotension

D) Nausea

E) Sweating

18. A 43-year-old woman on a camping trip has underestimated the

cool evening temperatures, so she wraps herself in a thin sheet of

polyester film with a reflective surface (Mylar), also known as an

emergency blanket or space blanket. She feels warm immediately.

Which heat loss mechanism most likely accounts for the

effectiveness of this paper-thin, reflective Mylar blanket?

A) Conduction to air

B) Conduction to objects

C) Convection

D) Evaporation

E) Radiation

Questions 19 and 20

Refer to the following figure to answer the next two questions.

19. Abundant amounts of adenosine triphosphate (ATP) in the

cytoplasm of the cell inhibit which step in glycolysis?

A) Conversion of glucose to glucose-6-phosphate

B) Conversion of fructose-6-phosphate to fructose-1,6-

diphosphate

C) Conversion of 1,3-diphosphoglyceric acid to 3-

phosphoglyceric acid

D) Conversion of phosphoenolpyruvic acid to pyruvic acid

20. Abundant amounts of adenosine diphosphate (ADP) or adenosine

monophosphate (AMP) stimulate which step in glycolysis?

A) Conversion of glucose to glucose-6-phosphate

B) Conversion of fructose-6-phosphate to fructose-1,6-

diphosphate

C) Conversion of 1,3-diphosphoglyceric acid to 3-

phosphoglyceric acid

D) Conversion of phosphoenolpyruvic acid to pyruvic acid

21. A 44-year-old woman with hepatic cirrhosis comes to her

physician for a checkup. Physical examination shows ascites. The

woman’s prothrombin time has doubled since her last visit 3

months ago, and her hematocrit is now 30%. What is the most

likely cause of this low hematocrit?

A) Colon cancer

B) Esophageal varices

C) Jaundice

D) Acute pancreatitis

E) Scleral icterus

22. During resting conditions, about 75% of the blood flowing

through the liver is from the portal vein, and the remainder is from

the hepatic artery. Which option best describes the liver circulation

in terms of resistance, pressure, and flow?

Resistance Pressure Flow

A) High High High

B) High Low High

C) Low High Low

D) Low Low High

E) Low Low Low

23. A scuba diver explores an underwater lava flow where the water

temperature is 102°F. Which profile best describes the mechanisms

of heat loss that are effective in this man?

Evaporation Radiation Convection Conduction

A) No No No Yes

B) No No No No

C) Yes Yes No Yes

D) No Yes No Yes

E) Yes Yes Yes Yes

24. A 34-year-old African American man is admied to the hospital

because of steadily increasing intense pain in the upper right side

of the abdomen. He is nauseated and vomiting. His hematocrit is

30. Ultrasonography shows the presence of gallstones. Which of

the following is the most likely major composition of the gallstones

in this man?

A) Bile pigments

B) Calcium carbonate

C) Calcium oxalate

D) Cholesterol

25. Deamination means removal of the amino groups from the amino

acids. Which substance is produced when deamination occurs by

transamination?

A) Acetyl coenzyme A

B) Ammonia

C) Citrulline

D) Ornithine

E) α-Ketoglutaric acid

26. Most of the energy released from a glucose molecule occurs as a

result of which process?

A) Citric acid cycle

B) Glycogenesis

C) Glycogenolysis

D) Glycolysis

E) Oxidative phosphorylation

27. A 32-year-old woman visits her physician because of loss of

appetite, fatigue, nausea, and dizziness. Physical examination

shows thinning hair. Blood tests show a hematocrit of 32. The

woman began following a vegetarian diet suggested by a friend 1

year ago. The physician suspects a dietary deficiently of which

substance?

A) Alanine

B) Glycine

C) Lysine

D) Serine

E) Tyrosine

28. A 45-year-old man is admied to the emergency department after

he was found lying in the street in an inebriated state. He is

markedly pale with icteric conjunctivae and skin. His abdomen is

distended, and he has shifting dullness, indicating ascites. His liver

is enlarged about 5 cm below the right costal margin and tender.

His spleen cannot be palpated. He has bilateral grade 2 edema of

the legs and feet. Which values of direct and indirect bilirubin (in

milligrams per deciliter) are most likely to be present in this man’s

plasma?

Direct Indirect

A) 1.1 1.2

B) 1.7 5.4

C) 2.4 2.5

D) 5.2 1.8

E) 5.8 7.2

Questions 29–31

The diagram shows the effects of changing the set-point temperature of

the hypothalamic temperature controller. The red line indicates the body

temperature, and the blue line represents the hypothalamic set-point

temperature. Use the figure to answer the next three questions.

29. Which set of changes occurs at point W compared with point V?

Shivering Sweating Vasoconstriction Vasodilation

A) No No No No

B) No Yes No Yes

C) No Yes Yes No

D) Yes No No Yes

E) Yes No Yes No

F) Yes Yes Yes Yes

30. Which set of changes occurs at point Y compared with point V?

Shivering Sweating Vasoconstriction Vasodilation

A) No No No No

B) No Yes No Yes

C) No Yes Yes No

D) Yes No No Yes

E) Yes No Yes No

F) Yes Yes Yes Yes

31. Which set of changes occurs at point X compared with point V?

Shivering Sweating Vasoconstriction Vasodilation

A) No No No No

B) No Yes No Yes

C) No Yes Yes No

D) Yes No No Yes

E) Yes No Yes No

F) Yes Yes Yes Yes

32. Which of the following is the most abundant source of highenergy phosphate bonds in the cells?

A) ATP

B) Phosphocreatine

C) ADP

D) Creatine

E) Creatinine

33. A 76-year-old man is admied to the emergency department after

being found lying in a field on a hot summer day. His body

temperature is 106°F, his blood pressure is normal, and his heart

rate is 160 beats/min. Which set of changes is most likely to be

present in this man?

Sweating Hyperventilation Vasodilation of Skin

A) No No No

B) No Yes Yes

C) Yes No Yes

D) Yes Yes No

E) Yes Yes Yes

34. Most of the energy for strenuous exercise that lasts for more than 5

to 10 seconds but less than 1 to 2 minutes comes from what source?

A) ATP

B) Anaerobic glycolysis

C) Oxidation of carbohydrates

D) Oxidation of lactic acid

E) Conversion of lactic acid into pyruvic acid

35. Erythrocytes are constantly dying and being replaced. Heme from

the hemoglobin is converted to what substance before being

eliminated from the body?

A) Bilirubin

B) Cholesterol

C) Cholic acid

D) Globin

E) Glucuronic acid

36. A 32-year-old pregnant woman in her third trimester is admied

to the emergency department because of severe upper right

quadrant pain after eating a meal of fried catfish. Her blood

pressure is 130/84 mm Hg, heart rate is 105 beats/min, and

respirations are 30/min. Her body mass index before pregnancy

was 45 kg/m2

. Physical examination shows abdominal guarding

and diaphoresis. Her serum bilirubin levels and white blood cell

count are both normal. Which of the following best describes this

patient’s condition?

A) Cholelithiasis

B) Constipation

C) Hepatitis

D) Pancreatitis

E) Peritonitis

37. An experimental device containing hepatocytes is developed to

provide effective support for patients with hepatic failure pending

liver transplantation. Hepatocyte viability is best documented by

an increase in which function?

A) Lactate dehydrogenase uptake

B) Ethanol output

C) Albumin output

D) Glucuronic acid uptake

E) Oxygen output

F) Carbon dioxide uptake

38. The metabolic rate of a person is typically expressed in terms of

the rate of heat liberation that results from the chemical reactions

of the body. Which factors tend to increase or decrease a person’s

metabolic rate?

Growth Hormone Fever Sleep Malnutrition

A) Decrease Decrease Decrease Decrease

B) Decrease Increase Decrease Increase

C)

Increase Increase Increase Increase

D)

Increase Increase Decrease Increase

E)

Increase Increase Decrease Decrease

39. An 8-year-old girl is taken to the physician because of diarrhea

and a red scaly rash. Physical examination shows mild cerebellar

ataxia. She is suspected of having pellagra because of these chronic

symptoms. However, she appears to be ingesting adequate

amounts of niacin in her diet, which is rich in meat. A brother has a

similar problem. Urine studies show large amounts of free amino

acids. Which diagnosis is most likely?

A) Alkaptonuria

B) Beriberi

C) Hartnup’s disease

D) Scurvy

E) Stickler’s syndrome

40. In a person with type 1 diabetes who is not receiving insulin

therapy and who has a fasting blood glucose of 400 mg/100 ml,

what would the respiratory quotient likely be 2 hours after eating a

light meal containing 60% carbohydrates, 20% protein, and 20%

fat?

A) 0.5

B) 0.7

C) 0.9

D) 1.0

E) 1.2

41. A 3-year-old white boy is extremely obese (weight of 37.5 kg), and

his parents report that he has a voracious appetite. What is the

most likely cause of his hyperphagia and obesity?

A) A lesion or destruction of the lateral hypothalamus

B) Excessive stimulation of the ventromedial nuclei of the

hypothalamus

C) A mutation that produces nonfunctional melanocortin-4

receptor protein

D) Excessive stimulation of pro-opiomelanocortin (POMC)

neurons

E) Excessive secretion of leptin

F) A mutation that prevents neuropeptide Y (NPY) formation in

hypothalamic neurons

42. Deficiency of which of the following would cause “night

blindness” in humans?

A) Vitamin A

B) Vitamin B1

C) Vitamin B6

D) Vitamin B12

E) Vitamin C

F) Niacin

43. Which changes would be expected to stimulate hunger in a person

who has not eaten for 24 hours?

A) Increased NPY in the hypothalamus

B) Increased leptin secretion

C) Increased peptide YY (PYY) secretion

D) Decreased ghrelin secretion

E) Activation of hypothalamic POMC neurons

F) Increased cholecystokinin secretion

44. Which of the following would be most important in contributing

to satiety after eating a large meal containing carbohydrates (50%),

fat (40%), and protein (10%)?

A) Release of cholecystokinin by the duodenum

B) Decreased leptin secretion

C) Increased release of endorphins

D) Increased ghrelin release by the stomach

E) Decreased release of PYY by the intestine

45. Deficiency of which vitamin is most likely to cause impaired blood

cloing?

A) Vitamin A

B) Vitamin B6

C) Vitamin C

D) Vitamin D

E) Vitamin K

46. Deficiency of which vitamin is the main cause of beriberi?

A) Vitamin A

B) Thiamine (vitamin B1

)

C) Riboflavin (vitamin B2

)

D) Vitamin B12

E) Pyridoxine (vitamin B6

)

47. Which of the following would tend to decrease hunger?

A) Increased release of endorphins

B) Increased ghrelin release by the stomach

C) Increased release of PYY by the intestine

D) Increased release of NPY by the hypothalamus

E) Increased release of cortisol by the adrenals

48. Which substance might be most useful in stimulating appetite in a

patient with cancer who has anorexia or cachexia?

A) Leptin

B) α-Melanocyte-stimulating hormone

C) PYY

D) Melanocortin-4 receptor antagonist

E) Ghrelin antagonist

F) Neuropeptide YY antagonist

49. The first stage in using triglycerides for energy is hydrolysis of the

triglycerides to which substances?

A) Acetyl coenzyme A and glycerol

B) Cholesterol and fay acids

C) Glycerol 3-phosphate and cholesterol

D) Glycerol and fay acids

E) Phospholipids and glycerol

50. Urinary nitrogen excretion measured in a patient is 16.0 g in 24

hours. What is the approximate amount of protein breakdown in

this patient for 24 hours in grams?

A) 16

B) 18

C) 100

D) 110

E) 120

Answers

1. E) During digestion, most triglycerides are split into monoglycerides

and fay acids. Then, while passing through the intestinal epithelial

cells, the monoglycerides and fay acids are resynthesized into new

molecules of triglycerides that enter the lymph as minute, dispersed

droplets called chylomicrons.

TMP14 pp. 853–854

2. A) The first stage in using triglycerides for energy is their hydrolysis into

fay acids and glycerol. Then, both the fay acids and the glycerol are

transported in the blood to the active tissues, where they will be

oxidized to give energy. Almost all cells—with some exceptions, such as

brain tissue and red blood cells—can use fay acids for energy.

TMP14 p. 856

3. A) Essentially all urea formed in the human body is synthesized in the

liver from ammonia. In the absence of the liver or in persons with

serious liver disease, ammonia accumulates in the blood. This

accumulation of ammonia is extremely toxic, especially to the brain, and

can lead to a state called hepatic coma or liver encephalopathy.

TMP14 pp. 869, 874

4. F) Bilirubin is formed during the degradation of red blood cells by

macrophages and released into plasma where it combines with albumin

(item 2). The free bilirubin is then taken up by hepatocytes (item 4),

where most of it is conjugated with glucuronic acid (item 1). The

conjugated bilirubin is then excreted from the hepatocytes by an active

transport process into the bile canaliculi and then into the intestines

(item 3).

TMP14 pp. 874–875

5. E) When liver parenchymal cells are destroyed, they are replaced with

fibrous tissue that eventually contracts around the blood vessels,

thereby greatly impeding the flow of portal blood through the liver. This

disease process is known as cirrhosis of the liver; it is a common

consequence of chronic alcoholism. The blockage of blood flow through

the liver raises the blood pressure in the upstream splanchnic organs,

causing ascites fluid to collect in the abdomen and varices to develop in

the lower esophagus. Peripheral edema can result from a failure of

hepatocytes to produce normal amounts of albumin (low colloid

osmotic pressure). Jaundice (hyperbilirubinemia) develops because of

damage to hepatocytes.

TMP14 pp. 871–872

6. E) This woman has acute pancreatitis caused by blockage of the papilla

of Vater by one or more gallstones. When a gallstone blocks the papilla

of Vater, the main secretory duct from the pancreas and the common

bile duct are blocked. The pancreatic enzymes are then dammed up in

the ducts and acini of the pancreas. Eventually, so much trypsinogen

accumulates that it overcomes the trypsin inhibitor in the secretions and

a small quantity of trypsinogen becomes activated to form trypsin.

When this happens, the trypsin activates still more trypsinogen, as well

as chymotrypsinogen and carboxypeptidase, resulting in a vicious circle

until most of the proteolytic enzymes in the pancreatic ducts and acini

become activated. These enzymes rapidly digest large portions of the

pancreas, sometimes completely and permanently destroying the ability

of the pancreas to secrete digestive enzymes.

TMP14 p. 835

7. C) Although a normal, unacclimatized person seldom produces more

than about 1 l of sweat per hour, when this person is exposed to hot

weather for 1 to 6 weeks, he or she begins to sweat more profusely, often

increasing maximum sweat production to as much as 2 to 3 l/hr. Also

associated with acclimatization is a further decrease in the concentration

of sodium chloride in the sweat, which allows progressively beer

conservation of body salt. Most of this effect is caused by increased

secretion of aldosterone by the adrenocortical glands, which results

from a slight decrease in sodium chloride concentration in the

extracellular fluid and plasma. An unacclimatized person who sweats

profusely often loses 15 to 30 g of salt each day for the first few days.

After 4 to 6 weeks of acclimatization, the loss is usually only 3 to 5

g/day.

TMP14 p. 911

8. D) The various methods by which heat is lost from the skin to the

surroundings include radiation, evaporation, and conduction to air and

conduction to objects. In a nude person siing inside at normal room

temperature, about 60% of total heat loss is by radiation, 22% by

evaporation, 15% by conduction to air, and 3% by conduction to objects.

TMP14 pp. 902–903

9. D) The set-point temperature in the hypothalamus is determined mainly

by the degree of activity of the heat temperature receptors in the

anterior hypothalamic- preoptic area. However, temperature signals

from the peripheral areas of the body, especially from the skin and

certain deep body tissues (spinal cord and abdominal viscera), also

contribute slightly to body temperature regulation. Many proteins,

breakdown products of proteins, and certain other substances,

especially lipopolysaccharide toxins released from bacterial cell

membranes, can cause the set-point of the hypothalamic thermostat to

rise. Substances that cause this effect are called pyrogens.

TMP14 pp. 909–910

10. B) The primary secretion of sweat is an active secretory product of the

epithelial cells lining the coiled portion of the sweat gland. Sympatheticcholinergic nerve fibers ending on or near the glandular cells elicit the

secretion. These are postganglionic sympathetic fibers that release

acetylcholine and hence are called sympathetic-cholinergic fibers; they

innervate sweat glands and some blood vessels.

TMP14 p. 904

11. E) When the body temperature has fallen below about 85°F, the ability

of the hypothalamus to regulate temperature is lost; it is greatly

impaired even when the body temperature falls below about 94°F. Part

of the reason for this diminished temperature regulation is that the rate

of chemical heat production in each cell is depressed almost 2-fold for

each 10°F decrease in body temperature. Also, sleepiness develops (later

followed by coma), which depresses the activity of the central nervous

system heat control mechanisms and prevents shivering.

TMP14 p. 911

12. A) Fay acids are degraded in mitochondria by the progressive release

of two-carbon segments in the form of acetyl coenzyme A. This process

is known as the beta-oxidation process for degradation of fay acids.

TMP14 pp. 856–857

13. B) The typical series of events that occur during a fever are shown in

the figure on the right. When pyrogens raise the set-point temperature

above its normal value, the body activates heat conservation and heat

production mechanisms that include cutaneous vasoconstriction,

piloerection, epinephrine secretion, and shivering. Within several

minutes, the body temperature increases to the elevated set-point value

of 103°F in this example. If the factor that is causing the high

temperature is removed, the hypothalamic set-point temperature

returns to a normal value of about 98.6°F, which leads to activation of

heat-loss mechanisms such as sweating and cutaneous vasodilation. The

body temperature then returns to its basal level.

TMP14 p. 910

14. B) This man has cirrhosis of the liver. Fluid accumulates in the

abdomen (ascites) for two main reasons: (1) decreased plasma colloid

osmotic pressure (COP) and (2) increased capillary hydrostatic pressure

in the splanchnic organs. The decrease in plasma COP results from

decreased production of albumin by liver hepatocytes; albumin

accounts for nearly 80% of the plasma COP. The low plasma COP also

promotes edema formation in the periphery, especially the feet. Liver

parenchymal cells are damaged or destroyed in persons with cirrhosis of

the liver. The cells are replaced with fibrous tissue that eventually

contracts around the blood vessels, thereby greatly impeding the flow of

portal blood through the liver. This increase in vascular resistance leads

to an increase in portal vein pressure, which in turn raises the capillary

hydrostatic pressure of the splanchnic organs. There is no reason to

assume that capillary hydrostatic pressure is also increased above

normal in the feet of this man.

TMP14 pp. 401, 867, 871

15. D) Evaporation is the only mechanism of heat loss from the body when

the environmental temperature is greater than the body temperature.

Each gram of water that evaporates from the surface of the body causes

0.58 kcal of heat to be lost from the body. Even when a person is not

sweating, water still evaporates insensibly from the skin and lungs at a

rate of 450 to 600 ml/day, which amounts to about 12 to 16 kcal of heat

loss per hour. Radiation, convection, and conduction are mechanisms of

heat loss when the body temperature is greater than the environmental

temperature.

TMP14 pp. 903–904

16. C) The metabolic rate increases after a meal because of various

chemical reactions associated with digestion, absorption, and storage of

food; this phenomenon is known as the thermogenic effect of food. After

a meal containing mostly carbohydrates and fats, the metabolic rate

usually increases by about 4%. However, a high-protein meal often

increases the metabolic rate by as much as 30%; this effect can last from

3 to 12 hours after the meal and is called the specific dynamic action of

proteins. Clearly, assimilation of proteins requires far more energy

expenditure compared with fats and carbohydrates.

TMP14 pp. 897–898

17. E) This man has heatstroke. When the body temperature rises into the

range of 105°F to 108°F, heatstroke likely follows. Heat loss mechanisms

are overwhelmed by excessive metabolic production of heat and

excessive environmental heat. Heatstroke is usually accompanied by

dehydration (poor skin turgor is common), which can produce nausea,

vomiting, hypotension, and fainting or dizziness. Interestingly, the skin

is frequently dry because the anterior hypothalamic-preoptic area of the

brain that normally initiates sweating is often compromised by the

elevation in body temperature.

TMP14 p. 911

18. E) Most of the heat loss from the body occurs by radiation in the form

of infrared heat waves, which is a type of electromagnetic wave. Heat

waves radiate from all objects toward the body, and the body radiates

heat waves to all surrounding objects. The reflective surface of the Mylar

blanket prevents heat loss by reflecting infrared heat waves from the

body back to the body, which causes the body to warm. At room

temperature, 60% of the heat loss occurs by radiation, 22% by

evaporation, 15% by conduction to air, and 3% by conduction to objects.

Convection (i.e., air currents) can increase heat loss by removing the

unstirred layer of air close to the skin.

TMP14 p. 903

19. B) Continual release of energy from glucose when energy is not needed

by the cells would be an extremely wasteful process. Both ATP and ADP

control the rate of chemical reactions in the energy metabolism

sequence. When ATP is abundant within the cell, it helps control energy

metabolism by inhibiting the conversion of fructose-6-phosphate to

fructose-1,6-diphosphate. It does so by inhibiting the enzyme

phosphofructokinase.

TMP14 p. 849

20. B) Both ADP and AMP increase the activity of the enzyme

phosphofructokinase and increase the conversion of fructose-6-

phosphate to fructose-1,6-diphosphate.

TMP14 p. 849

21. B) Esophageal varices are extremely dilated submucosal veins in the

lower third of the esophagus. The submucosal veins have a normal

diameter of about 1 mm and can enlarge to 1 to 2 cm with prolonged

portal hypertension, which is common in persons with cirrhosis of the

liver. The presence of ascites indicates that the patient has portal

hypertension. The dilated esophageal veins often bleed and thus lower

the hematocrit. Although colon cancers can also bleed, there is no reason

to assume colon cancer in this woman. Pancreatitis can occur in persons

with chronic alcoholism, but there is no evidence for this condition, and

substantial bleeding is not common in persons with pancreatitis.

Jaundice and scleral icterus (i.e., yellowing of the sclera) are common in

persons with cirrhosis, but these conditions are unlikely to cause

significant bleeding.

TMP14 pp. 871–872

22. D) The liver has a high blood flow, low vascular resistance, and low

blood pressure. During resting conditions, about 27% of the cardiac

output flows through the liver, yet the pressure in the portal vein

leading into the liver averages only 9 mm Hg. This high flow and low

pressure indicate that the resistance to blood flow through the hepatic

sinusoids is normally very low.

TMP14 p. 871

23. B) None of the mechanisms of heat loss is effective when a person is

placed in water that has a temperature greater than body temperature.

Instead, the body will continue to gain heat until the body temperature

becomes equal to the water temperature.

TMP14 p. 903

24. A) This man has sickle cell disease, which is a hemolytic disease that

results in the premature destruction of red blood cells. Release of

hemoglobin from damaged red blood cells leads to high levels of

bilirubin in the blood plasma. This increase in bilirubin can lead to the

development of pigment stones in the gallbladder that are composed

primarily of bilirubin.

TMP14 pp. 446, 875

25. B) The degradation of amino acids occurs almost entirely in the liver,

and it begins with deamination, which occurs mainly by the following

transamination schema: The amino group from the amino acid is

transferred to α-ketoglutaric acid, which then becomes glutamic acid.

The glutamic acid then transfers the amino group to still other

substances or releases it in the form of ammonia. In the process of losing

the amino group, the glutamic acid again becomes α-ketoglutaric acid,

so that the cycle can repeat again and again.

TMP14 pp. 868–869

26. E) About 90% of the total ATP produced by glucose metabolism is

formed during oxidation of the hydrogen atoms released during the

early stages of glucose degradation. This process is called oxidative

phosphorylation. Only two ATP molecules are formed by glycolysis, and

another two are formed in the citric acid cycle. ATP is not formed by

glycogenesis or glycogenolysis.

TMP14 p. 848

27. C) Lysine is an essential amino acid, which means that it must be

included in the diet because the body cannot synthesize it. Alanine,

glycine, serine, and tyrosine can be synthesized by the body and are

therefore considered nonessential amino acids. This woman has a lysine

deficiency, which is common in poorly designed vegetarian diets;

symptoms include nausea, fatigue, dizziness, anemia, loss of appetite,

and thinning hair. Good dietary sources of lysine include eggs, meat,

beans, legumes, soy, dairy products, and certain fish (such as cod and

sardines). L-lysine is a building block for all proteins in the body.

TMP14 pp. 866, 868

28. D) This man has cirrhosis of the liver. In this condition, the rate of

bilirubin production is normal, and the free bilirubin still enters the liver

cells and becomes conjugated in the usual way. The conjugated bilirubin

(direct) is mostly returned to the blood, probably by rupture of

congested bile canaliculi, so that only small amounts enter the bile. The

result is elevated levels of conjugated (direct) bilirubin in the plasma,

with normal or near-normal levels of unconjugated (indirect) bilirubin.

TMP14 pp. 871, 874–875

29. E) When the hypothalamic set-point temperature is greater than the

body temperature, the person feels cold and exhibits responses that lead

to an elevation of body temperature. These responses include shivering

and vasoconstriction, as well as piloerection and epinephrine secretion.

Shivering increases heat production. The increase in epinephrine

secretion causes an immediate increase in the rate of cellular

metabolism, which is an effect called chemical thermogenesis.

Vasoconstriction of the skin blood vessels decreases heat loss through

the skin.

TMP14 p. 910

30. B) When the hypothalamic set-point temperature is lower than the

body temperature, the person feels hot and exhibits responses that cause

body temperature to decrease. These responses include sweating and

vasodilation. Sweating increases heat loss from the body by evaporation.

Vasodilation of skin blood vessels facilitates heat loss from the body by

increasing the skin blood flow.

TMP14 p. 910

31. A) When the hypothalamic set-point temperature is equal to the body

temperature, the body exhibits neither heat loss nor heat conservation

mechanisms, even when the body temperature is far above normal.

Therefore, the person does not feel hot even when the body temperature

is 104°F.

TMP14 p. 910

32. B) Phosphocreatine contains high-energy phosphate bonds and is three

to eight times as abundant as ATP or ADP in a cell. Creatine does not

contain high-energy phosphate bonds. Creatinine is a breakdown

product of creatine phosphate in muscle.

TMP14 p. 894

33. B) This patient has heatstroke. Patients with heatstroke commonly

exhibit tachypnea and hyperventilation caused by direct central nervous

system stimulation, acidosis, or hypoxia. The blood vessels in the skin

are vasodilated, and the skin is warm. Sweating ceases in patients with

true heatstroke, most likely because the high temperature itself causes

damage to the anterior hypothalamic-preoptic area. The nerve impulses

from this area are transmied in the autonomic pathways to the spinal

cord and then through sympathetic outflow to the skin to cause

sweating.

TMP14 p. 911

34. B) Most of the extra energy required for strenuous activity that lasts for

more than 5 to 10 seconds but less than 1 to 2 minutes is derived from

anaerobic glycolysis. Release of energy by glycolysis occurs much more

rapidly than oxidative release of energy, which is much too slow to

supply the needs of the muscle in the first few minutes of exercise. ATP

and phosphocreatine already present in the cells are rapidly depleted in

less than 5 to 10 seconds. After the muscle contraction is over, oxidative

metabolism is used to reconvert much of the accumulated lactic acid

into glucose; the remainder becomes pyruvic acid, which is degraded

and oxidized in the citric acid cycle.

TMP14 p. 894

35. A) Hemoglobin is metabolized by tissue macrophages (also called the

reticuloendothelial system). The hemoglobin is first split into globin and

heme, and the heme ring is opened to produce free iron and a straight

chain of four pyrrole nuclei, from which bilirubin will eventually be

formed. The free bilirubin is taken up by hepatic cells, and most of it is

conjugated with glucuronic acid; the conjugated bilirubin passes into the

bile canaliculi and then into the intestines.

TMP14 p. 875

36. A) Cholelithiasis is the presence of gallstones (choleliths) in the

gallbladder or bile ducts. This patient exhibits typical symptoms caused

by gallstones.

TMP14 p. 820

37. C) Hepatocytes produce essentially all the albumin normally present in

blood. Viable hepatocytes use oxygen and produce carbon dioxide.

Glucuronic acid produced by hepatocytes is used to conjugate bilirubin,

forming bilirubin glucuronide. Lactate dehydrogenase is an enzyme that

converts pyruvic acid to lactic acid under anaerobic conditions.

TMP14 pp. 317, 873–874

38. E) Growth hormone can increase the metabolic rate 15% to 20% as a

result of direct stimulation of cellular metabolism. Fever, regardless of

its cause, increases the chemical reactions of the body by an average of

about 120% for every 10°C rise in temperature. The metabolic rate

decreases 10% to 15% below normal during sleep. Prolonged

malnutrition can decrease the metabolic rate 20% to 30%, presumably

because of the paucity of food substances in the cells.

TMP14 p. 898

39. C) This child has Hartnup’s disease. This condition resembles pellagra

(because of the symptoms of diarrhea, dementia, and dermatitis) and

may be misdiagnosed as a nutritional deficiency of niacin. Hartnup’s

disease is an autosomal-recessive trait caused by a defective gene that

codes for a sodium-dependent and chloride-independent neutral amino

acid transporter expressed mainly in kidney and intestinal epithelium.

Poor epithelial transport of neutral amino acids (such as tryptophan)

leads to poor absorption of dietary amino acids, as well as excess amino

acid excretion in the urine. Tryptophan is a precursor of niacin; it is an

essential amino acid that must be included in the diet. Alkaptonuria,

also called “black urine disease,” is a genetic disorder of phenylalanine

and tyrosine metabolism. Beriberi is caused by a nutritional deficit in

thiamine. Scurvy results from a deficiency of vitamin C, which is

required for collagen synthesis. Stickler’s syndrome is a group of genetic

disorders that affect connective tissues; it is characterized by eye

problems, hearing loss, joint problems, and facial abnormalities.

TMP14 p. 889

40. B) Type 1 diabetes is characterized by a lack of insulin. In the absence

of adequate insulin, lile carbohydrate can be used by the body’s cells,

and the respiratory quotient remains near that for fat metabolism (0.70).

TMP14 p. 879

41. C) Mutations that produce a nonfunctional melanocortin-4 receptor

cause extreme obesity and may account for as much as 5% to 6% of early

onset, morbid obesity in children. All the other changes would tend to

reduce food intake and/or increase energy expenditure and thus cause

weight loss rather than obesity.

TMP14 pp. 880–881

42. A) One of the basic functions of vitamin A is in the formation of retinal

pigments and therefore the prevention of night blindness.

TMP14 p. 888

43. A) NPY is an orexigenic neurotransmier that stimulates feeding and is

increased during food deprivation. Leptin, PYY, cholecystokinin, and

activation of POMC neurons are all reduced by fasting. Ghrelin is

increased, not decreased, by fasting.

TMP14 pp. 880–883

44. A) Cholecystokinin is released mainly in response to fats and proteins

entering the duodenum and activates sensory receptors in the

duodenum, sending messages to the brain stem via vagal afferents that

contribute to satiation and meal cessation. All the other changes would

tend to increase rather than decrease food intake.

TMP14 p. 882

45. E) Vitamin K is an essential co-factor to a liver enzyme that adds a

carboxyl group to factors II (prothrombin), VII (proconvertin), IX, and X,

all of which are important to blood coagulation. The other vitamins

listed are not directly involved in coagulation.

TMP14 p. 891

46. B) Thiamine is needed for the final metabolism of carbohydrates and

amino acids. Decreased utilization of these nutrients secondary to

thiamine deficiency is responsible for many of the characteristics of

beriberi, including peripheral vasodilation and edema, lesions of the

central and peripheral nervous system, and gastrointestinal tract

disturbances.

TMP14 pp. 888–889

47. C) PYY is released from most parts of the intestinal tract, but especially

from the ileum and colon, in response to food intake. Increased levels of

PYY have been shown to decrease food intake. All the other changes

tend to increase food intake.

TMP14 pp. 880–881

48. D) Antagonists of melanocortin-4 receptors have been shown to

markedly aenuate anorexia (i.e., reduced food intake due to decreased

appetite) and cachexia (i.e., increased energy expenditure as well as

decreased food intake) by blocking hypothalamic melanocortin-4

receptors. All the other choices would tend to decrease appetite and/or

increase energy expenditure, exacerbating the anorexia or cachexia of a

patient with cancer.

TMP14 p. 880–881, 887

49. D) Triglycerides are hydrolyzed to glycerol and fay acids, which, in

turn, are oxidized to provide energy. Almost all cells, with the exception

of some brain tissue, can use fay acids almost interchangeably with

glucose for energy.

TMP14 pp. 856–857

50. D) The rate of protein metabolism can be estimated by measuring the

nitrogen in the urine, then adding 10 percent (about 90% of the nitrogen

in proteins is excreted in the urine) and multiplying by 6.25 (100/16)

because the average protein contains about 16% nitrogen.

TMP14 p. 879


Unit XIV: Endocrinology and

Reproduction

1. Which of the following is expected to exhibit the greatest biological

activity?

A) Insulin like growth factor-1 free in the plasma

B) Cholecalciferol (vitamin D3)

C) Cortisol bound to corticosteroid binding globulin

D) T4 bound to thyroxine binding globulin

E) Aldosterone bound to plasma albumin

2. Which receptor controls nitric oxide (NO) release to cause

vasodilation during penile erection?

A) Leptin receptor

B) Angiotensin AT1 receptor

C) Endothelin ETA receptor

D) Muscarinic receptor

3. After menopause, hormone replacement therapy with estrogen-like

compounds is effective in preventing the progression of

osteoporosis. What is the mechanism of their protective effect?

A) They stimulate the activity of osteoblasts

B) They increase absorption of calcium from the gastrointestinal

tract

C) They stimulate calcium reabsorption by the renal tubules

D) They stimulate parathyroid hormone (PTH) secretion by the

parathyroid gland

4. Neurons that secrete antidiuretic hormone or oxytocin terminate in

which of the following structures?

A) Posterior pituitary

B) Median eminence

C) Mammillary body

D) Paraventricular nucleus

E) Supraoptic nucleus

5. Which of the following represents a physiological action of growth

hormone?

A) Increases the breakdown of muscle protein

B) Increases utilization of glucose in muscle

C) Decreases storage of lipids in adipose cells

D) Decreases gene transcription

E) Decreases gluconeogenesis in the liver

6. Which hormones antagonize the effect of NO and cause the penis to

become flaccid after orgasm?

A) Endothelin and norepinephrine

B) Estrogen and progesterone

C) Luteinizing hormone (LH) and follicle-stimulating hormone

(FSH)

D) Progesterone and LH

Questions 7–9


The red lines in the above figure illustrate the normal relationships

between plasma insulin concentration and glucose production in the liver

and between plasma insulin concentration and glucose uptake in muscle.

Use this figure to answer Questions 7–9.

7. Which lines most likely illustrate these relationships in a patient

with type 2 diabetes?

A) A and C

B) A and D

C) B and C

D) B and D

8. Which lines most likely illustrate these relationships in a patient

with acromegaly?

A) A and C

B) A and D

C) B and C

D) B and D

9. Line D most likely illustrates the influence of which of the

following?

A) Exercise

B) Obesity

C) Growth hormone (GH)

D) Cortisol

E) Glucagon

10. Thecal cells in the follicle are not able to produce what sex steroid?

A) Estradiol

B) Testosterone

C) Progesterone

D) Dihydrotestosterone

11. A baby is born with a penis, a scrotum with no testes, no vagina,

and XX chromosomes. This condition is referred to as

hermaphroditism. What could cause this abnormality?

A) Abnormally high levels of human chorionic gonadotropin

(hCG) production by the trophoblast cells

B) The presence of a testosterone-secreting tumor in the mother’s

right adrenal gland

C) Abnormally high levels of LH in the maternal blood

D) Abnormally low levels of testosterone in the maternal blood

E) Abnormally low rates of estrogen production by the placenta

12. Antidiuretic hormone (ADH) is increased by which of the

following?

A) A hyperosmotic extracellular fluid in the hypothalamus

B) A hyperosmotic extracellular fluid in the adenohypophysis

C) A hypoosmotic extracellular fluid in the hypothalamus

D) A hypoosmotic extracellular fluid in the adenohypophysis

E) A hypoosmotic fluid in the atria of the heart

13. In an individual with panhypopituitarism, which selection below

best describes the plasma hormone changes that would occur?

A) ↓GHRH, ↓somatostatin, ↓growth hormone, ↓somatomedin C

B) ↓GHRH, ↓somatostatin, ↓growth hormone, ↑somatomedin C

C) ↑GHRH, ↑somatostatin, ↑growth hormone, ↓somatomedin C

D) ↑GHRH, ↑somatostatin, ↓ growth hormone, ↓somatomedin C

E) ↑GHRH, ↓somatostatin, ↓growth hormone, ↓somatomedin C

14. Which of the following could inhibit the initiation of labor?

A) Administration of an antagonist of the actions of

progesterone

B) Administration of LH

C) Administration of an antagonist of PGE2 effects

D) Mechanically dilating and stimulating the cervix

E) Administration of oxytocin

15. A patient has nephrogenic diabetes insipidus. Which of the

following would either be expected or a suggested intervention?

A) Decreased plasma sodium concentration

B) Increased secretion of ADH from the supraoptic nuclei

C) High urine osmolality

D) Increased AVPR2 function

E) Decrease secretion of ADH from the supraoptic and

paraventricular nuclei

16. Which of the following would most likely cause a decrease in the

release of thyroid-stimulating hormone?

A) Decreased iodinase enzyme

B) Decreased iodine pump activity in thyroid gland

C) Decreased body temperature

D) Increased thyrotropin releasing hormone

E) Increased plasma thyroxine by venous infusion

17. The increased cardiac output caused by elevated circulating levels

of thyroid hormones is most likely caused by

A) Direct actions of thyroid-stimulating hormone on the heart

y g

B) Direct actions of thyroid-stimulating hormone on the brain

C) An increase in the metabolic demand of the tissues

D) An increase in plasma cholesterol and triglycerides

E) An increase in total body weight

18. If a radioimmunoassay is properly conducted and the amount of

radioactive hormone bound to antibody is low, what would this

result indicate?

A) Plasma levels of endogenous hormone are high

B) Plasma levels of endogenous hormone are low

C) More antibody is needed

D) Less radioactive hormone is needed

19. Which of the following depicts the most likely sequence of events

in an individual exposed to cold?

A) ↑Thyrotropin-releasing hormone, ↑thyroid-stimulating

hormone, ↑thyroxine

B) ↑Thyrotropin-releasing hormone, ↓thyroid-stimulating

hormone, ↑thyroxine

C) ↑Thyroid-stimulating hormone, ↑thyrotropin-releasing

hormone, ↑thyroxine

D) ↑Thyroid-stimulating hormone, ↓thyrotropin-releasing

hormone, ↑thyroxine

E) ↑Thyroxine, ↑thyrotropin-releasing hormone, ↑thyroidstimulating hormone

20. Spermatogenesis is regulated by a negative feedback control

system in which FSH stimulates the steps in sperm cell formation.

Which negative feedback signal associated with sperm cell

production inhibits pituitary formation of FSH?

A) Testosterone

B) Inhibin

C) Estrogen

D) LH

21. In an individual with a thyroid hormone producing adenoma, one

might expect which of the following?

A) ↑ T4, ↓ T3, ↓ TRH, ↓ TSH

B) ↑ T4, ↑ T3, ↓ TRH, ↓ TSH

C) ↑ T4, ↑ T3, ↑ TRH, ↓ TSH

D) ↑ T4, ↑ T3, ↓ TRH, ↑ TSH

E) ↓ T4, ↑ T3, ↓ TRH, ↓ TSH

22. When do progesterone levels rise to their highest point during the

female hormonal cycle?

A) Between ovulation and the beginning of menstruation

B) Immediately before ovulation

C) When the blood concentration of LH is at its highest point

D) When 12 primary follicles are developing to the antral stage

23. You suspect thyroid disease in a female patient. Based on the

plasma values below, which of the following would be expected?

[TSH] Total [T4] [TBG]

Normal range

0.4-5.5 mU/l 5.6-14.7 µg/dl 1.7-3.6 µg/dl

Patient data

9.3 2.3 3.0

A) Graves’ disease

B) Secondary hyperthyroidism

C) Hashimoto’s disease

D) Secondary hypothyroidism

E) Euthyroid pregnant

24. Which of the following enzymes catalyzes the conversion of

cholesterol to pregnenolone?

A) Aldosterone synthase

B) Lipoprotein lipase

C) Hormone sensitive lipase

D) 11β-Hydroxylase

E) Cholesterol desmolase

25. Which of the following would most likely occur if plasma

aldosterone levels were low?

A) Hyperkalemia

B) Hypokalemia

C) Hypernatremia

D) Hypertension

26. A professional athlete in her mid-20s has not had a menstrual

cycle for 5 years, although a bone density scan revealed normal

skeletal mineralization. Which fact may explain these

observations?

A) She consumes a high-carbohydrate diet

B) Her grandmother sustained a hip fracture at age 79 years

C) Her blood pressure is higher than normal

D) Her plasma estrogen concentration is very low

E) She has been taking anabolic steroid supplements for 5 years

27. During a chronic infusion of aldosterone in an experimental

animal model, one would expect which of the following?

A) ↑Blood pressure, ↔extracellular fluid volume, ↓urinary

sodium excretion

B) ↑Blood pressure, ↓extracellular fluid volume, ↔urinary

sodium excretion

C) ↑Blood pressure, ↔extracellular fluid volume, ↑urinary

sodium excretion

D) ↑Blood pressure, ↑extracellular fluid volume, ↔urinary

sodium excretion

E) ↑Blood pressure, ↔extracellular fluid volume, ↔urinary

sodium excretion

28. In the circulatory system of a fetus, which of the following is

greater before birth than after birth?

A) Arterial Po2

B) Right atrial pressure

C) Aortic pressure

D) Left ventricular pressure

29. In response to a physiological stimulus such as the stress of taking

an important quiz, which of the following reflects the most likely

sequence of events?

A) ↑Cortisol, ↑corticotropin, ↑corticotropin-releasing hormone

B) ↑Corticotropin-releasing hormone, ↑corticotropin, ↑cortisol

C) ↑Cortisol, ↓corticotropin, ↑corticotropin-releasing hormone

D) ↑Corticotropin-Releasing hormone, ↑corticotropin, ↓cortisol

E) ↑Cortisol, ↑corticotropin, ↓corticotropin-releasing hormone

30. Which of the following best characterizes the metabolic actions of

cortisol?

A) ↑Muscle glucose uptake, ↑muscle amino acid uptake,

↑adipose tissue fat uptake

B) ↑Muscle glucose uptake, ↓muscle amino acid uptake, ↑adipose

tissue fat uptake

C) ↓Muscle glucose uptake, ↓muscle amino acid uptake,

↑adipose tissue fat uptake

D) ↓Muscle glucose uptake, ↑muscle amino acid uptake,

↓adipose tissue fat uptake

p p

E) ↓Muscle glucose uptake, ↓muscle amino acid uptake, ↓adipose

tissue fat uptake

31. Which of the following is most likely to occur as a result of chronic

hyperglycemia associated with untreated type 1 diabetes mellitus?

A) Increased intracellular fluid volume

B) Decreased urinary glucose

C) Metabolic alkalosis

D) Osmotic diuresis and polyuria

E) Improved eyesight

32. Which enzyme in the cytochrome P450 steroid synthesis cascade is

directly responsible for estradiol synthesis?

A) 17-Beta-hydroxysteroid dehydrogenase

B) 5-Alpha reductase

C) Aromatase

D) Side chain cleavage enzyme

33. Which of the following is greater after birth than before birth?

A) Flow through the foramen ovale

B) Pressure in the right atrium

C) Flow through the ductus arteriosus

D) Aortic pressure

34. Immediately after consuming a meal consisting of a large burger,

French fries, onion rings, and a diet cola, one might expect a

DECREASE in which of following?

A) Amino acid transport into cells

B) Fay acid synthesis

C) Hormone sensitive lipase

D) Liver glycogen

E) Cell permeability to glucose

35. In an individual with untreated insulin dependent diabetes

mellitus (type 1), one would expect which of the following?

A) ↑Plasma free fay acids, ↓liver glycogen, ↑skeletal muscle

mass

B) ↑Plasma free fay acids, ↓liver glycogen, ↓skeletal muscle

mass

C) ↑Plasma free fay acids, ↑liver glycogen, ↓skeletal muscle

mass

D) ↓Plasma free fay acids, ↓liver glycogen, ↑skeletal muscle

mass

E) ↓Plasma free fay acids, ↑liver glycogen, ↓skeletal muscle

mass

36. Which of the following changes would be expected to help

maintain plasma glucose in the postabsorptive?

A) ↓Insulin, ↑glucagon, ↓growth hormone, ↓cortisol

B) ↓Insulin, ↑glucagon, ↑growth hormone, ↓cortisol

C) ↓Insulin, ↑glucagon, ↑growth hormone, ↑cortisol

D) ↑Insulin, ↓glucagon, ↓growth hormone, ↑cortisol

E) ↑Insulin, ↓glucagon, ↑growth hormone, ↑cortisol

37. For male differentiation to occur during embryonic development,

testosterone must be secreted from the testes. What stimulates the

secretion of testosterone during embryonic development?

A) LH from the maternal pituitary gland

B) hCG

C) Inhibin from the corpus luteum

D) GnRH from the embryo’s hypothalamus

38. Which of the following best describes insulin?

A) Lipid-soluble hormone tightly bound to plasma proteins

B) Peptide hormone that activates an intracellular receptor

C) Peptide hormone that activates a G-coupled protein receptor

D) Peptide hormone that activates an enzyme-linked receptor

E) Steroid hormone that activates an enzyme-linked receptor

39. If one were to experience a sudden decrease in extracellular fluid

calcium, which of the following would most likely be the first

physiological response to buffer the change in calcium?

A) Increased calcium absorption in the gut

B) Decreased phosphate absorption in the gut

C) Increased parathyroid hormone from the anterior pituitary

D) Decreased renal excretion of phosphate

E) Increased exchange of calcium with the bone fluid

40. As menstruation ends, estrogen levels in the blood rise rapidly.

What is the source of the estrogen?

A) Corpus luteum

B) Developing follicles

C) Endometrium

D) Stromal cells of the ovaries

E) Anterior pituitary gland

41. A 30-year-old woman reports to the clinic for a routine physical

examination. The examination reveals she is pregnant. Her plasma

p g p

levels of TSH are high, but her total thyroid hormone concentration

is normal. Which of the following best reflects the patient’s clinical

state?

A) Graves’ disease

B) Hashimoto’s disease

C) A pituitary tumor secreting TSH

D) A hypothalamic tumor secreting thyrotropin-releasing

hormone (TRH)

E) The patient is taking thyroid extract

42. Which of the following would be expected in a patient with

chronic renal failure?

Plasma[1,25-(OH)2D] Plasma [PTH] Bone Resorption

A) ↑ ↑ ↑

B) ↑ ↑ ↓

C) ↑ ↓ ↓

D) ↓ ↓ ↑

E) ↓ ↑ ↓

F) ↓ ↑ ↑

43. A female athlete who took testosterone-like steroids for several

months stopped having normal menstrual cycles. What is the best

explanation for this observation?

A) Testosterone stimulates inhibin production from the corpus

luteum

B) Testosterone binds to receptors in the endometrium, resulting

in the failure of the endometrium to develop during the

normal cycle

C) Testosterone binds to receptors in the anterior pituitary that

stimulate the secretion of FSH and LH

D) Testosterone inhibits the hypothalamic secretion of GnRH

and the pituitary secretion of LH and FSH

44. An experiment is conducted in which ADH is administered at

hour 3 to four subjects (A to D). In the above figure, which lines

most likely reflect the response to ADH administration in a normal

patient and in a patient with central diabetes insipidus?

Normal Central Diabetes Insipidus

A) B A

B) B D

C) D A

D) D B

45. Which of the following decreases the resistance in the arteries

leading to the sinuses of the penis?

A) Stimulation of the sympathetic nerves innervating the arteries

B) NO

C) Inhibition of activity of the parasympathetic nerves leading to

the arteries

D) All the above

46. Using the three following statements, select the best answer.

1. Hydroxyapatite is the major crystalline salt in calcified bone

2. An osteon is made up of concentric layers of bone called

lamellae

3. Osteocytes are the major cells responsible for the formation of

new bone tissue

A) Only statement 1 is correct

B) Statement 1 and 2 are correct

C) Statements 1 and 3 are correct

D) All statements are correct

E) No statements are correct

47. All of the following statements about parathyroid hormone are

true EXCEPT one. Which one is the EXCEPTION?

A) PTH directly activates osteoblasts and osteocytes

B) PTH inhibits the production of vitamin D hormones

C) PTH promotes bone resorption in response to decreased

plasma calcium

D) PTH promotes the movement of calcium from bone fluid to

the extracellular fluid

E) PTH promotes calcium reabsorption in the renal distal tubule

and collecting duct

48. A 46-year-old man has “puffy” skin and is lethargic. His plasma

TSH concentration is low and increases markedly when he is given

TRH. What is the most likely diagnosis?

A) Hyperthyroidism due to a thyroid tumor

B) Hyperthyroidism due to an abnormality in the hypothalamus

C) Hypothyroidism due to an abnormality in the thyroid

D) Hypothyroidism due to an abnormality in the hypothalamus

E) Hypothyroidism due to an abnormality in the pituitary

49. Negative feedback on FSH release from the anterior pituitary in

men that results in a reduction in estradiol production is due to

which hormone?

A) Progesterone

B) Estradiol

C) Testosterone

D) Inhibin

50. During the first few years after menopause, FSH levels are

normally extremely high. A 56-year-old woman completed

menopause 3 years ago. However, she is found to have low levels

of FSH in her blood. What is the best explanation for this finding?

A) She has been receiving hormone replacement therapy with

estrogen and progesterone since she completed menopause

B) Her adrenal glands continue to produce estrogen

C) Her ovaries continue to secrete estrogen

D) She took birth control pills for 20 years before menopause

51. Blockade of what receptors will prolong erection in a man?

A) Estrogen receptors

B) Cholesterol receptors

C) Muscarinic receptors

D) Phosphodiesterase-5 receptors

52. Which of the following pairs of hormones and the corresponding

action is incorrect?

A) Glucagon—increased glycogenolysis in liver

B) Glucagon—increased glycogenolysis in skeletal muscle

C) Glucagon—increased gluconeogenesis

D) Cortisol—increased gluconeogenesis

E) Cortisol—decreased glucose uptake in muscle

53. A large dose of insulin is administered intravenously to a patient.

Which set of hormonal changes is most likely to occur in the

plasma in response to the insulin injection?

Growth Hormone Glucagon Epinephrine

A) ↑ ↓ ↔

B) ↔ ↑ ↑

C) ↑ ↑ ↑

D) ↓ ↑ ↑

E) ↓ ↓ ↔

54. What is a frequent cause of delayed breathing at birth?

A) Fetal hypoxia during the birth process

B) Maternal hypoxia during the birth process

C) Fetal hypercapnia

D) Maternal hypercapnia

55. Which hormone is largely unbound to plasma proteins?

A) Cortisol

B) T4

C) ADH

D) Estradiol

E) Progesterone

56. What is the mechanism by which the zona pellucida becomes

“hardened” after penetration of a sperm cell to prevent a second

sperm from penetrating?

A) A reduction in estradiol

B) The proteins released from the acrosome of the sperm

C) An increase in intracellular calcium in the oocyte

D) An increase in testosterone that affects the sperm

57. Why is milk produced by a woman only after delivery, not before?

A) Levels of LH and FSH are too low during pregnancy to

support milk production

B) High levels of progesterone and estrogen during pregnancy

suppress milk production

C) The alveolar cells of the breast do not reach maturity until

after delivery

D) High levels of oxytocin are required for milk production to

begin, and oxytocin is not secreted until the baby stimulates

the nipple

58. Which of the following increases the rate of excretion of calcium

ions by the kidney?

A) A decrease in calcitonin concentration in the plasma

B) An increase in phosphate ion concentration in the plasma

C) A decrease in the plasma level of PTH

D) Metabolic alkalosis

59. A patient has hyperthyroidism due to a pituitary tumor. Which set

of physiological changes would be expected?

Thyroglobulin Synthesis Heart Rate Exophthalmos

A) ↑ ↑ +

B) ↑ ↑ −

C) ↑ ↓ +

D) ↓ ↓ +

E) ↓ ↓ −

F) ↓ ↑ −

60. A 25-year-old man is severely injured when hit by a speeding

vehicle and loses 20% of his blood volume. Which set of

physiological changes would be expected to occur in response to

the hemorrhage?

Atrial Stretch Receptor Activity Arterial Baroreceptor Activity ADH Secretion

A) ↓ ↓ ↑

B) ↓ ↓ ↓

C) ↔ ↑ ↑

D) ↑ ↑ ↑

E) ↑ ↑ ↓

61. If a woman has a tumor that is secreting large amounts of estrogen

from the adrenal gland, which of the following will occur?

A) Progesterone levels in the blood will be very low

B) Her LH secretion rate will be totally suppressed

C) She will not have normal menstrual cycles

D) Her bones will be normally calcified

E) All the above

62. When compared with the postabsorptive state, which set of

metabolic changes would most likely occur during the

postprandial state?

Hepatic Glucose Uptake Muscle Glucose Uptake Hormone-Sensitive Lipase Activity

A) ↑ ↑ ↑

B) ↑ ↓ ↑

C) ↓ ↑ ↓

D) ↑ ↑ ↓

E) ↓ ↑ ↑

63. Very early in embryonic development, testosterone is formed

within male embryos. What is the function of this hormone at this

stage of development?

A) Stimulation of bone growth

B) Stimulation of development of male sex organs

C) Stimulation of development of skeletal muscle

D) Inhibition of LH secretion

64. During spermatogenesis, estrogen is produced by

A) Leydig cells in response to FSH

B) Sertoli cells in response to FSH

C) Leydig cells in response to LH

D) Sertoli cells in response to LH

65. A patient arrives in the emergency department apparently in

cardiogenic shock due to a massive heart aack. His initial arterial

blood sample reveals the following concentrations of ions and pH

level:

Sodium 137 mmol/l

Bicarbonate

14 mmol/l

Free calcium 2.8 mmol/l

Potassium 4.8 mmol/l

pH 7.16

To correct the acidosis, the aending physician begins an infusion of

sodium bicarbonate and after 1 hour obtains another blood sample, which

reveals the following values:

Sodium 138 mmol/l

Bicarbonate

22 mmol/l

Free calcium 2.3 mmol/l

Potassium 4.5 mmol/l

pH 7.34

What is the cause of the decrease in calcium ion concentration?

A) The increase in arterial pH resulting from the sodium

bicarbonate infusion inhibited PTH secretion

B) The increase in pH resulted in the stimulation of osteoblasts,

which removed calcium from the circulation

C) The increase in pH resulted in an elevation in the concentration

of HPO4 −, which shifted the equilibrium between HPO4 − and

Ca2+

 toward CaHPO4

D) The increase in arterial pH stimulated the formation of 1,25-

dihydroxycholecalciferol, which resulted in an increased rate of

absorption of calcium from the gastrointestinal tract

66. The prostate fluid contributes the bulk of the volume of semen,

which includes

A) Calcium, citrate, phosphate, and profibrinolysin

B) Fructose, citric acid, prostaglandins, and fibrinogen

C) Sex hormones

D) Mucus

67. A 30-year-old woman is breastfeeding her infant. During suckling,

which hormonal response is expected in the woman?

A) Increased secretion of ADH from the supraoptic nuclei

B) Increased secretion of ADH from the paraventricular nuclei

C) Increased secretion of oxytocin from the paraventricular

nuclei

D) Decreased secretion of neurophysin

E) Increased plasma levels of both oxytocin and ADH

68. A 30-year-old man has Conn’s syndrome. Which set of

physiological changes is most likely to occur in this patient

compared with a healthy person?

Arterial Pressure Extracellular Fluid Volume Sodium Excretion

A) ↔ ↔ ↔

B) ↑ ↔ ↔

C) ↑ ↑ ↔

D) ↔ ↑ ↓

E) ↑ ↑ ↓

69. Which of the following is important in the process of capacitation

of sperm after ejaculation?

A) Microtubule reorganization

B) Increased testosterone secretion by spermatozoa

C) Washout of inhibitory factors

D) Influx of glucose

70. Dehydroepiandrosterone sulfate (DHEAS), the precursor for the

high levels of estradiol that occur in pregnancy, is made in what

tissue?

A) Fetal adrenal gland

B) Ovary of the mother

C) Placenta

D) Adrenal gland of the mother

71. What is the consequence of sporadic nursing of the neonate by the

mother?

A) An increase in prolactin-releasing hormone

B) An increase in oxytocin

C) Lack of birth control

D) Lack of prolactin surge

72. Which of the following would be associated with parallel changes

in aldosterone and cortisol secretion?

A) Addison’s disease

B) Cushing’s disease

C) Cushing’s syndrome (adrenal tumor)

D) A low-sodium diet

E) Administration of a converting enzyme inhibitor

73. The process of spermatogenesis begins with spermatogonia and

results in which of the following?

A) 1 diploid spermatid

B) 4 diploid spermatids

C) 1 haploid spermatid

D) 2 haploid spermatids

E) 4 haploid spermatids

74. RU486 causes abortion if it is administered before or soon after

implantation. What is the specific effect of RU486?

A) It binds to LH receptors, stimulating the secretion of

progesterone from the corpus luteum

B) It blocks progesterone receptors so that progesterone has no

effect within the body

C) It blocks the secretion of FSH by the pituitary

D) It blocks the effects of oxytocin receptors in the uterine

muscle

75. A 55-year-old man has developed the syndrome of inappropriate

antidiuretic hormone secretion due to carcinoma of the lung.

Which physiological response would be expected?

A) Increased plasma osmolality

B) Inappropriately low urine osmolality (relative to plasma

osmolality)

C) Increased thirst

D) Decreased secretion of ADH from the pituitary gland

p y g

76. During pregnancy, the uterine smooth muscle is quiescent. During

the ninth month of gestation, the uterine muscle becomes

progressively more excitable. What factor contributes to the

increase in excitability?

A) Placental estrogen synthesis rises to high rates

B) Progesterone synthesis by the placenta decreases

C) Uterine blood flow reaches its highest rate

D) PGE2 synthesis by the placenta decreases

E) Activity of the fetus falls to low levels

77. A 20-year-old woman is not having menstrual cycles. Her plasma

progesterone concentration is found to be minimal. What is the

explanation for the low level of progesterone?

A) LH secretion rate is elevated

B) LH secretion rate is suppressed

C) FSH secretion rate is suppressed

D) No corpus luteum is present

E) High inhibin concentration in the plasma has suppressed

progesterone synthesis

78. Before the preovulatory surge in LH, granulosa cells of the follicle

secrete which hormone?

A) Testosterone

B) Progesterone

C) Estrogen

D) Inhibin

Questions 79 and 80

79. Based on the above figure, which set of curves most likely reflects

the responses in a healthy individual and in patients with type 1 or

type 2 diabetes mellitus (Diabetes mellitus)?

Healthy Type 1 Diabetes mellitus Type 2 Diabetes mellitus

A) 3 2 1

B) 1 2 3

C) 1 3 2

D) 2 1 3

E) 2 3 1

80. Based on the above figure, which set of curves most likely reflects

the responses in a healthy person and in a patient in the early

stages of Cushing’s syndrome?

Healthy Cushing’s Syndrome

A) 3 2

B) 1 2

C) 1 3

D) 2 1

E) 2 3

81. Which hormone activates enzyme-linked receptors?

A) ADH

B) Insulin

C) ACTH

D) PTH

E) Aldosterone

82. Which of the following is produced by the trophoblast cells during

the first 3 weeks of pregnancy?

A) Estrogen

B) LH

C) Oxytocin

D) hCG

E) None of the above

83. Which of the following is higher in a neonate than in a fetus?

A) Flow through the foramen ovale

B) Right atrial pressure

C) Flow through the ductus arteriosus

D) Aortic pressure

84. Which finding is most likely in a patient who has myxedema?

A) Somnolence

B) Palpitations

C) Increased respiratory rate

D) Increased cardiac output

E) Weight loss

85. At birth, a large, well-nourished baby is found to have a plasma

glucose concentration of 17 mg/dl (normal is 80 to 100 mg/dl) and a

plasma insulin concentration twice the normal value. What is the

explanation for these findings?

A) The neonate experienced in utero malnutrition

B) The mother was malnourished during pregnancy

C) The mother has diabetes, with poorly controlled

hyperglycemia

D) The mother is obese

86. Degradation of the corpus luteum is prevented by which of the

following?

A) Increased estrogen secretion by the developing placenta

B) Release of hCG from the trophoblasts

C) Forward positive regulation by LH

D) Placental derived prolactin

87. Which of the following stimulates the secretion of PTH?

A) An increase in extracellular calcium ion activity above the

normal value

B) An increase in calcitonin concentration

C) Respiratory acidosis

D) Increased secretion of PTH-releasing hormone from the

hypothalamus

E) None of the above

88. A 40-year-old woman consumes a high-potassium diet for several

weeks. Which hormonal change is most likely to occur?

A) Increased secretion of DHEA

B) Increased secretion of cortisol

C) Increased secretion of aldosterone

D) Increased secretion of ACTH

E) Decreased secretion of CRH

89. After implantation into the uterus, nutrition of the blastocyst

comes from which structure?

A) Placenta

B) Decidua

C) Glomerulosa cells

D) Corpus luteum

90. Which hormone is not stored in its endocrine-producing gland?

A) T4

B) PTH

C) Aldosterone

D) ACTH

E) Insulin

91. A young woman comes to the emergency department with a

vertebral compression fracture. Radiographs of the spine indicate

generalized demineralization. She is vegetarian, does not smoke or

drink alcohol, and has a normal plasma potassium concentration of

5.4 mEq/l, a sodium concentration of 136 mEq/l, and a plasma

calcium concentration of 7.0 mg/dl. Her vitamin D3

 value is several

times greater than normal, although her 1,25-

dihydroxycholecalciferol concentration is at the lower limit of

detectability. She has been in renal failure for the past 5 years and

undergoes hemodialysis three times each week. What is the cause

of her low 1,25-dihydroxycholecalciferol level?

A) Metabolic acidosis

B) Metabolic alkalosis

C) She is unable to form 1,25-dihydroxycholecalciferol because

of her extensive kidney disease

D) She is undergoing dialysis with a dialysis fluid that does not

contain calcium

E) She is taking receiving calcium supplements

92. A placenta is incapable of synthesizing which hormones?

A) Estrogen

B) Progesterone

C) Androgens

D) Estriol

93. Which of the following hormones is most closely associated with

the secretory phase of the endometrial cycle?

A) Progesterone

B) Estrogen

C) FSH

D) LH

E) Inhibin

94. Which finding would likely be reported in a patient with a

deficiency in iodine intake?

A) Weight loss

B) Nervousness

C) Increased sweating

D) Increased synthesis of thyroglobulin

E) Tachycardia

95. A 37-year-old woman presents to her physician with an enlarged

thyroid gland and high plasma levels of T4

 and T3

. Which of the

following is likely to be decreased?

A) Heart rate

B) Cardiac output

C) Peripheral vascular resistance

D) Ventilation rate

E) Metabolic rate

96. Before intercourse, a woman irrigates her vagina with a solution

that lowers the pH of the vaginal fluid to 4.5. What will be the

effect on sperm cells in the vagina?

A) The metabolic rate will increase

B) The rate of movement will decrease

C) The formation of PGE2 will increase

D) The rate of oxygen consumption will increase

97. Which hormonal responses would be expected after a meal high in

protein?

Insulin Glucagon Growth Hormone

A) ↑ ↑ ↓

B) ↑ ↑ ↑

C) ↑ ↓ ↓

D) ↓ ↓ ↑

E) ↓ ↑ ↑

98. Men who take large doses of testosterone-like androgenic steroids

for long periods are sterile in the reproductive sense of the word.

What is the explanation for this finding?

A) High levels of androgens bind to testosterone receptors in the

Sertoli cells, resulting in overstimulation of inhibin formation

B) Overstimulation of sperm cell production results in the

formation of defective sperm cells

C) High levels of androgen compounds inhibit the secretion of

GnRH by the hypothalamus, resulting in the inhibition of LH

and FSH release by the anterior pituitary

D) High levels of androgen compounds produce hypertrophic

dysfunction of the prostate gland

99. Cortisone is administered to a 30-year-old woman for the

treatment of an autoimmune disease. Which of the following is

most likely to occur?

A) Increased ACTH secretion

B) Increased cortisol secretion

C) Increased insulin secretion

D) Increased muscle mass

E) Hypoglycemia between meals

100. In the hypothalamic-pituitary-gonadal axis of the female, what is

the follicular cell type that produces inhibin?

A) Cytotrophoblasts

B) Synthiotrophoblasts

C) Granulosa

D) Thecal

101. The function of which of the following is increased by an elevated

parathyroid hormone concentration?

A) Osteoclasts

B) Hepatic formation of 25-hydroxycholecalciferol

C) Phosphate reabsorptive pathways in the renal tubules

D) All the above

102. Which statement about peptide or protein hormones is usually

true?

A) They have longer half-lives than steroid hormones

B) They have receptors on the cell membrane

C) They have a slower onset of action than both steroid and

thyroid hormones

D) They are not stored in endocrine-producing glands

103. Which set of physiological changes would be most likely to occur

in a patient with acromegaly?

Pituitary Mass Kidney Mass Femur Length

A) ↓ ↓ ↑

B) ↓ ↑ ↑

C) ↑ ↔ ↔

D) ↑ ↑ ↔

E) ↑ ↑ ↑

104. Cortisol and GH are most dissimilar in their metabolic effects on

which of the following?

A) Protein synthesis in muscle

B) Glucose uptake in peripheral tissues

C) Plasma glucose concentration

D) Mobilization of triglycerides

105. Why do infants of mothers who had adequate nutrition during

pregnancy not require iron supplements or a diet rich in iron until

about 3 months of age?

A) Growth of the infant does not require iron until after the third

month

B) The fetal liver stores enough iron to meet the infant’s needs

until the third month

C) Synthesis of new red blood cells begins after 3 months

D) Muscle cells that develop before the third month do not

contain myoglobin

106. Cortisone is administered to a patient for the treatment of an

autoimmune disease. Which of the following would least likely

occur in response to the cortisone treatment?

A) Hypertrophy of the adrenal glands

B) Increased plasma levels of C-peptide

C) Decreased CRH secretion

D) Increased blood pressure

E) Hyperglycemia

107. All of the following accurately describe the regulation of the

female sexual cycle EXCEPT one. Which one is the EXCEPTION?

A) Estradiol inhibits GnRH release during the post ovulatory

phase

B) Progesterone increases GnRH release during the post

ovulatory phase

C) Estradiol increases LH in the days immediately preceding

ovulation

D) Falling progesterone and estrogen late in the luteal phase

allows LH and FSH to rise

E) LH and FSH increase estradiol release during the follicular

phase

108. If a male is born without a penis and testes, a defect is likely in

which gene on the Y chromosome?

A) ERE—estrogen response element

B) ARE—androgen response element

C) SRY—affecting Sertoli cells

D) ERG—early response genes

y p g

109. Where does fertilization normally take place?

A) Uterus

B) Cervix

C) Ovary

D) Ampulla of the fallopian tubes

110. Which finding is most likely to occur in a patient who has

uncontrolled type 1 diabetes mellitus?

A) Decreased plasma osmolality

B) Increased plasma volume

C) Increased plasma pH

D) Increased release of glucose from the liver

E) Decreased rate of lipolysis

111. GH secretion would most likely be suppressed under which

condition?

A) Acromegaly

B) Gigantism

C) Deep sleep

D) Exercise

E) Acute hyperglycemia

112. Pregnenolone is not in the biosynthetic pathway of which

substance?

A) Cortisol

B) Estrogen

C) Aldosterone

D) 1,25(OH)2D

E) DHEA

113. Two days before the onset of menstruation, secretions of FSH and

LH reach their lowest levels. What is the cause of this low level of

secretion?

A) The anterior pituitary gland becomes unresponsive to the

stimulatory effect of GnRH

B) Estrogen from the developing follicles exerts a feedback

inhibition on the hypothalamus

C) The rise in body temperature inhibits hypothalamic release of

GnRH

D) Secretion of estrogen, progesterone, and inhibin by the corpus

luteum suppresses hypothalamic secretion of GnRH and

pituitary secretion of FSH

114. Which condition contributes to “sodium escape” in persons with

Conn’s syndrome?

A) Decreased plasma levels of atrial natriuretic peptide

B) Increased plasma levels of angiotensin II

C) Decreased sodium reabsorption in the collecting tubules

D) Increased arterial pressure

115. Which of the following most accurately describes events in the

female sexual cycle?

A) FSH causes the development of the corpus luteum

B) Estrogen and LH have a positive feedback relationship during

the late follicular phase

C) Estrogens are primarily produced by theca cells in the

developing ovary

D) During the luteal phase, estrogen increases to a greater degree

than progesterone

E) LH is most responsible for the development of primary

follicles

116. A 30-year-old woman reports to the clinic for a routine physical

examination, which reveals she is pregnant. Her plasma levels of

TSH are high, but her total T4

 concentration (protein bound and

free) is normal. Which of the following best reflects this patient’s

clinical state?

A) Graves’ disease

B) Hashimoto’s disease

C) A pituitary tumor that is secreting TSH

D) A hypothalamic tumor that is secreting TRH

E) The patient is taking thyroid extract

117. A man has a disease that destroyed only the motor neurons of the

spinal cord below the thoracic region. Which aspect of sexual

function would not be possible?

A) Arousal

B) Erection

C) Lubrication

D) Ejaculation

118. Which of the following is responsible for invasion of the uterus

and formation of the placenta?

A) Trophoblasts

B) Oocyte

C) Decidua

D) Endometrium

119. A sustained program of lifting heavy weights will increase bone

mass. What is the mechanism of this effect of weightlifting?

A) Elevated metabolic activity stimulates parathyroid hormone

secretion

B) Mechanical stress on the bones increases the activity of

osteoblasts

C) Elevated metabolic activity results in an increase in dietary

calcium intake

D) Elevated metabolic activity results in stimulation of calcitonin

secretion

120. The hormone most responsible for maintaining milk production

after parturition is

A) Estrogen

B) Progesterone

C) Oxytocin

D) Prolactin

E) Inhibin

121. Which of the following would be expected in a patient with a

genetic deficiency of 11-β-hydroxysteroid dehydrogenase type II?

A) Hyperkalemia

B) Hypertension

C) Increased plasma renin activity

D) Increased plasma [aldosterone]

E) Hyperglycemia

122. Which physiological response is greater for T3

 than for T4?

A) Secretion rate from the thyroid

B) Plasma concentration

C) Plasma half-life

D) Affinity for nuclear receptors in target tissues

E) Latent period for the onset of action in target tissues

123. A “birth control” compound for men has been sought for several

decades. Which substance would provide effective sterility?

A) A substance that mimics the actions of LH

B) A substance that blocks the actions of inhibin

C) A substance that blocks the actions of FSH

D) A substance that mimics the actions of GnRH

124. For milk to flow from the nipple of the mother into the mouth of

the nursing infant, what must occur?

A) Myoepithelial cells must relax

B) Prolactin levels must fall

C) Oxytocin secretion from the posterior pituitary must take

place

D) The baby’s mouth must develop a strong negative pressure

over the nipple

E) All the above

125. A number of normal physiological changes occur during

pregnancy. Which of the following best describes one of these

changes in the mother?

A) Increase total peripheral resistance

B) Increased cardiac output

C) Decreased metabolic rate

D) Decreased body weight

E) Decreased uterine size

126. Which set of physiological changes would be expected in a

nondiabetic patient with Cushing’s disease?

Plasma Aldosterone Plasma Cortisol Plasma Insulin

A) ↑ ↑ ↑

B) ↑ ↑ ↔

C) ↑ ↔ ↔

D) ↔ ↔ ↑

E) ↔ ↑ ↔

F) ↔ ↑ ↑

127. When compared with the late-evening values typically observed

in normal subjects, plasma levels of both ACTH and cortisol would

be expected to be higher in which persons?

A) Normal subjects after waking in the morning

B) Normal subjects who have taken dexamethasone

C) Patients with Cushing’s syndrome (adrenal adenoma)

D) Patients with Addison’s disease

E) Patients with Conn’s syndrome

128. Which of the following conditions or hormones would most

likely increase GH secretion?

A) Hyperglycemia

B) Exercise

C) Somatomedin

D) Somatostatin

E) Aging

129. Which set of findings would be expected in a person maintained

on a long-term low-sodium diet?

Plasma [Aldosterone] Plasma [Atrial Natriuretic Peptide] Plasma [Cortisol]

A) ↑ ↑ ↔

B) ↑ ↓ ↓

C) ↑ ↓ ↔

D) ↔ ↔ ↔

E) ↓ ↓ ↓

F) ↓ ↑ ↓

130. What would be associated with parallel changes in aldosterone

and cortisol secretion?

A) Addison’s disease

B) Cushing’s disease

C) Cushing’s syndrome (ectopic ACTH-producing tumor)

D) A high-sodium diet

E) Administration of a converting enzyme inhibitor

131. Which blood vessel in the fetus has the highest PO 2?

A) Ductus arteriosus

B) Ductus venosus

C) Ascending aorta

D) Left atrium

132. A 59-year-old woman has osteoporosis, hypertension, hirsutism,

and hyperpigmentation. Magnetic resonance imaging indicates

that the pituitary gland is not enlarged. Which condition is most

consistent with these findings?

A) Pituitary ACTH-secreting tumor

B) Ectopic ACTH-secreting tumor

C) Inappropriately high secretion rate of CRH

D) Adrenal adenoma

E) Addison’s disease

133. Which set of findings is an inappropriate hypophysial hormone

response to the hypothalamic hormone listed?

Hypothalamic Hormone Secretion Hypophysial Hormone

A) Somatostatin ↓ GH

B) Dopamine ↑ Prolactin

C) GnRH ↑ LH

D) TRH ↑ TSH

E) CRH ↑ ACTH

134. A patient is administered sufficient T4

 to increase plasma levels of

the hormone several fold. Which set of changes is most likely in

this patient after several weeks of T4

 administration?

Respiratory Rate Heart Rate Plasma Cholesterol Concentration

A) ↑ ↑ ↑

B) ↑ ↑ ↓

C) ↑ ↓ ↑

D) ↓ ↓ ↑

E) ↓ ↑ ↓

135. Which of the following hormones is most critical for sustaining a

successful pregnancy, even up through week 12 of gestation?

A) Estrogen

B) Progesterone

C) hCG

D) GnRH

E) Inhibin

136. What causes menopause?

A) Reduced levels of gonadotropic hormones secreted from the

anterior pituitary gland

B) Reduced responsiveness of the follicles to the stimulatory

effects of gonadotropic hormones

C) Reduced rate of secretion of progesterone from the corpus

luteum

D) Reduced numbers of follicles available in the ovary for

stimulation by gonadotropic hormones

137. What does not increase when insulin binds to its receptor?

A) Fat synthesis in adipose tissue

B) Protein synthesis in muscle

C) Glycogen synthesis

D) Gluconeogenesis in the liver

E) Intracellular tyrosine kinase activity

138. Release of which hormone is an example of neuroendocrine

secretion?

F) GH

G) Cortisol

H) Oxytocin

I) Prolactin

J) ACTH

139. The ability of a fetus to effectively use the relatively low maternal

Po2 is facilitated by which of the following?

A) Decreased glucose transport in the placental villi

B) Increased production of amniotic fluid

C) Increased total fetal hemoglobin concentration

D) Decreased placental membrane permeability

E) Decreased fetal hemoglobin binding capacity

140. Inhibition of the iodide pump would be expected to cause which

change?

A) Increased synthesis of T4

B) Increased synthesis of thyroglobulin

C) Increased metabolic rate

D) Decreased TSH secretion

E) Extreme nervousness

141. Before implantation, the blastocyst obtains its nutrition from

uterine endometrial secretions. How does the blastocyst obtain

nutrition during the first week after implantation?

A) It continues to derive nutrition from endometrial secretions

B) The cells of the blastocyst contain stored nutrients that are

metabolized for nutritional support

C) The placenta provides nutrition derived from maternal blood

D) The trophoblast cells digest the nutrient-rich endometrial cells

and then absorb their contents for use by the blastocyst

142. Which pituitary hormone has a chemical structure most similar

to that of ADH?

A) Oxytocin

B) ACTH

C) TSH

D) FSH

E) Prolactin

143. Which option would not be efficacious in the treatment of

patients with type 2 diabetes?

A) Glucocorticoids

B) Insulin injections

C) Thiazolidinediones

D) Sulfonylureas

E) Weight loss

144. Which of the following is most likely to occur in the early stages

of type 2 diabetes?

A) Increased insulin sensitivity

B) Decreased hepatic glucose output

C) Increased plasma levels of C-peptide

D) Increased plasma [β-hydroxybutyric acid]

E) Hypovolemia

145. What is the most common cause of respiratory distress syndrome

in neonates born at 7 months’ gestation?

A) Pulmonary edema due to pulmonary arterial hypertension

B) Formation of a hyaline membrane over the alveolar surface

C) Failure of the alveolar lining to form adequate amounts of

surfactant

D) Excessive permeability of the alveolar membrane to water

146. Which of the following is an expected circulatory change that

occurs after birth?

A) Opening of the ductus venosus

B) Opening of the foramen ovale

C) Opening of the ductus arteriosus

D) Closing of the ductus arteriosus

E) Closing of the inferior vena cava

147. A 45-year-old woman has a mass in the sella turcica that

compresses the portal vessels, disrupting pituitary access to

p p p g p y

hypothalamic secretions. The secretion rate of which hormone

would most likely increase in this patient?

A) ACTH

B) GH

C) Prolactin

D) LH

E) TSH

148. Which of the following is not produced by osteoblasts?

A) Alkaline phosphatase

B) RANK ligand

C) Collagen

D) Pyrophosphate

E) Osteoprotegerin

149. Which set of findings would be expected in a patient with

primary hyperparathyroidism?

Plasma [1,25-(OH)2D3] Plasma [Phosphate] Urinary Ca

2+ Excretion

A) ↑ ↑ ↑

B) ↑ ↓ ↑

C) ↑ ↓ ↓

D) ↓ ↓ ↑

E) ↓ ↑ ↓

F) ↓ ↑ ↑

150. A man who has been exposed to high levels of gamma radiation

is sterile due to destruction of the germinal epithelium of the

seminiferous tubules, although he has normal levels of

testosterone. Which of the following would be found in this

patient?

A) A normal secretory paern of GnRH

B) Normal levels of inhibin

C) Suppressed levels of FSH

D) Absence of Leydig cells

Questions 151 and 152

An experiment was conducted in which rats were injected with one of

two hormones or saline solution (control) for 2 weeks. Autopsies were then

performed, and organ weights were measured (in milligrams). Use this

information to answer Questions 151 and 152.

Control Hormone 1 Hormone 2

Pituitary

12.9 8.0 14.5

Thyroid 250 500 245

Adrenal glands 40 37 85

Body weight 300 152 175

151. What is hormone 1?

A) TRH

B) TSH

C) T4

D) ACTH

E) Cortisol

152. What is hormone 2?

A) TSH

B) T4

C) CRH

D) ACTH

E) Cortisol

153. An infant is born with yellow pigmentation in the skin and eyes.

What is the most likely cause of this?

A) Elevated circulating ACTH

B) Poor renal clearance of bilirubin

C) The lack of a fully functional liver at birth

D) Failure of the foramen ovale to close

E) Delayed onset of breathing

Answers

1. A) The freely circulating (unbound) hormone is the biologically active

hormone. Cholecalciferol is a prohormone and thus is not the

biologically active vitamin D hormone. In this question, cortisol, T4, and

aldosterone are all bound to carrier proteins.

TMP14 p. 929

2. D) Parasympathetic postganglionic fibers release acetylcholine that

activates muscarinic receptors on endothelium to produce NO and

increases cyclic guanosine monophosphate, which activates protein

kinase G, causing a reduction in intracellular calcium (also increasing

NO by positive feedback) and causing vasodilation.

TMP14 p. 1027

3. A) Estrogen compounds are believed to have an osteoblast-stimulating

effect. When the amount of estrogen in the blood falls to very low levels

after menopause, the balance between the bone-building activity of the

osteoblasts and the bone-degrading activity of the osteoclasts is tipped

toward bone degradation. When estrogen compounds are added as part

of hormone replacement therapy, the bone-building activity of the

osteoblasts is increased to balance the osteoclastic activity.

TMP14 pp. 949, 1045

4. A) ADH is made in the supraoptic nuclei of the hypothalamus. It is

transported in nerve fibers along with neurophysin carrier proteins that

pass through the pituitary stalk and terminate in the posterior pituitary.

TMP14 pp. 948–949

5. C) GH promotes several metabolic changes. These include a net increase

in amino acid uptake in the muscle and liver, a decrease in glucose

utilization and storage, and an increase in lipolysis. The net effect of GH

is to decrease glucose and lipid storage in adipose cells.

TMP14 pp. 943–944

6. A) Norepinephrine is released from the nerve terminals, and endothelin

is released from endothelial cells in the vasculature, causing

vasoconstriction of the vasculature.

TMP14 p. 1027

7. C) Type 2 diabetes mellitus is characterized by diminished sensitivity of

target tissues to the metabolic effects of insulin—that is, there is insulin

resistance. As a result, hepatic uptake of glucose is impaired, and

glucose release is enhanced. In muscle, the uptake of glucose is

impaired.

TMP14 pp. 985–986, 995

8. C) In acromegaly, high plasma levels of GH cause insulin resistance.

Consequently, glucose production by the liver is increased, and glucose

uptake by peripheral tissues is impaired.

TMP14 pp. 943–944, 996–997

9. A) During exercise, glucose utilization by muscle is increased, which is

largely independent of insulin.

TMP14 p. 985

10. A) Thecal cells do not have the capacity to produce estradiol because

they lack aromatase.

TMP14 pp. 1040, 1043, 1044

11. B) A very high concentration of testosterone in a female embryo will

induce formation of male genitalia. An adrenal tumor in the mother that

synthesizes testosterone at a high, uncontrolled rate could produce the

masculinizing effect.

TMP14 pp. 1043, 1044

12. A) Osmoreceptors in, or near, the hypothalamus are important

regulators of ADH. Hyperosmotic extracellular fluid causes the cells of

the hypothalamus to shrink and stimulates the release of ADH, which

promotes renal H2O reabsorption to restore the extracellular fluid to

isosmotic.

TMP14 p. 949

13. E) An individual with panhypopituitarism has generalized dysfunction

of the pituitary gland. GHRH from the hypothalamus is increased in an

aempt to restore the pituitary function. For similar reasons,

somatostatin is decreased. Because pituitary function is impaired,

growth hormone production is reduced, and because growth hormone

stimulates the production of somatomedin, its production is also

reduced.

TMP14 pp. 946–947

14. C) Antagonism of progesterone’s effects, dilation of the cervix, and

oxytocin all increase uterine smooth muscle excitability and facilitate

contractions and the onset of labor. LH would have no effect.

Prostaglandin E2

 strongly stimulates uterine smooth muscle contraction

and is formed at an increasing rate by the placenta late in gestation.

TMP14 pp. 1064, 1066

15. B) AVPR2 function is impaired in patients with nephrogenic diabetes

insipidus, rendering ADH ineffective at increasing H2O reabsorption in

the distal nephron. This causes a compensatory increase in the release of

ADH from the supraoptic nuclei of the hypothalamus. Patients with

diabetes insipidus run the risk of developing hypernatremia, and they

produce a large volume of dilute urine.

TMP14 pp. 381, 439

16. E) Thyroxine (T4) is the major thyroid hormone, along with

triiodothyronine (T3). An increase in the thyroid hormones aenuates

the production of thyroid-stimulating hormone (TSH) through negative

feedback inhibition.

TMP14 pp. 958–959

17. C) Thyroid hormones cause a general increase in basal metabolic rate.

With an increased metabolic rate, there is an increased metabolic

demand of the tissues which is the primary determinant of cardiac

output.

TMP14 p. 957

18. A) In a radioimmunoassay, there is too lile antibody to completely

bind the radioactively tagged hormone and the hormone in the fluid

(plasma) to be assayed. Thus, there is competition between the labeled

and endogenous hormone for binding sites on the antibody.

Consequently, if the amount of radioactive hormone bound to antibody

is low, this finding would indicate that plasma levels of endogenous

hormone are high.

TMP14 p. 936

19. A) Cold exposure is an important physiological stimulus for the

production and release of the thyroid hormones. Cold causes the

hypothalamic production of thyrotropin-releasing hormone, which

stimulates thyrotropes of the anterior pituitary to release thyroidstimulating hormone (TSH). The increased TSH stimulates the

production of the thyroid hormones, including thyroxine, which helps

to relieve the physiological stress caused by the cold.

TMP14 pp. 958–959

20. B) The Sertoli cells of the seminiferous tubules secrete inhibin at a rate

proportional to the rate of production of sperm cells. Inhibin has a direct

inhibitory effect on anterior pituitary secretion of FSH. FSH binds to

specific receptors on the Sertoli cells, causing the cells to grow and

secrete substances that stimulate sperm cell production. The secretion of

inhibin thereby provides the negative feedback control signal from the

seminiferous tubules to the pituitary gland.

TMP14 p. 1033

21. B) A thyroid hormone–producing adenoma causes an increase in

thyroid hormones. Thus, one would expect an increase in both

circulating T4 and T3 caused by the adenoma. The increased T4 and T3

feeds back to inhibit the production and release of TRH from the

hypothalamus and TSH from the anterior pituitary to halt further

production of the thyroid hormones. However, the adenoma does not

respond to normal feedback regulation, and thus T3 and T4 remain

high.

TMP14 pp. 958–961

22. A) The corpus luteum is the only source of progesterone production,

except for minute quantities secreted from the follicle before ovulation.

The corpus luteum is functional between ovulation and the beginning of

menstruation, during which time the concentration of LH is suppressed

below the level achieved during the preovulatory LH surge.

TMP14 pp. 1046–1047

23. C) The high levels of TSH (outside the normal range) are indicative of

hypofunction of the thyroid, and this is further observed with the low

total T4. Thyroxine-binding globulin remains in the normal range,

making the best answer Hashimoto’s disease, which is the most

common form of hypothyroidism. Secondary hypothyroidism occurs in

response to failure of the pituitary gland to stimulate the thyroid.

Therefore, the high TSH rules out this possibility.

TMP14 pp. 961–962

24. E) Cholesterol desmolase is the key enzyme responsible for the

conversion of cholesterol to pregnenolone for the process of steroid

synthesis.

TMP14 p. 966

25. A) Aldosterone increases the Na+K+ ATPase in the basolateral

membrane of the principal cells and increases ENaC channels in the

luminal side. This creates a driving force for Na+ reabsorption and K+

excretion leading to hypokalemia. When aldosterone is low, K+

excretion is aenuated, leading to hyperkalemia.

TMP14 pp. 969–970

26. E) Anabolic steroids bind to testosterone receptors in the

hypothalamus, providing feedback inhibition of normal ovarian cycling

and preventing menstrual cycling as well as stimulation of osteoblastic

activity in the bones.

TMP14 pp. 1028, 1031

27. D) Chronically elevated aldosterone increases sodium and water

retention leading to an expansion of extracellular fluid volume.

Increased extracellular fluid leads to increased blood pressure, which

promotes pressure natriuresis, causing urinary sodium excretion to

come into balance. Thus, during a chronic infusion urinary sodium

excretion is not changed.

TMP14 p. 970

28. B) Right atrial pressure falls dramatically after the onset of breathing

because of a reduction in pulmonary vascular resistance, pulmonary

arterial pressure, and right ventricular pressure.

TMP14 pp. 1073–1075

29. B) Physiological stimuli for glucocorticoids, such as stress, cause the

hypothalamic production of corticotropin-releasing hormone (CRH).

CRH stimulates corticotropes from the anterior pituitary to release

corticotropin (or ACTH). Corticotropin promotes the production of

cortisol from the adrenal cortex to help alleviate the physiological

stressor.

TMP14 pp. 974–977

30. E) The metabolic actions of cortisol increase the availability of

circulating fuel sources in response to physiological stressors. Cortisol

impairs skeletal muscle glucose and amino acid uptake (although it

promotes hepatic amino acid uptake) and promotes lipolysis from

adipocytes. This has the net effect to increase plasma glucose, free fay

acids, and amino acids.

TMP14 pp. 972–973

31. D) Glucose is normally filtered in the glomerulus and reabsorbed in the

proximal tubule. However, during untreated type I diabetes, the amount

of filtered glucose exceeds (180 mg/dl) the reabsorptive capacity of the

proximal tubule, increasing urinary osmolarity. This causes an increase

in water filtration, leading to frequent urination (polyuria).

TMP14 pp. 995

32. C) Aromatase causes conversion of testosterone to estradiol.

TMP14 p. 1043

33. D) Because of the loss of blood flow through the placenta, systemic

vascular resistance doubles at birth, which increases the aortic pressure

as well as the pressure in the left ventricle and left atrium.

TMP14 pp. 1073, 1074

34. C) Consuming a meal consisting of carbohydrate, protein, and fat will

stimulate the production and release of insulin, which promotes energy

storage. Insulin increases cell permeability to glucose to promote its

storage in the form of glycogen (liver) and fat through fay acid

synthesis and storage in the adipose. Hormone-sensitive lipase

promotes the breakdown of fat to free fay acids and is decreased in

response to insulin.

TMP14 pp. 985–989

35. B) Type I diabetes is associated with low insulin and thus an impaired

ability to store energy. Thus, in the absence of insulin, plasma free fay

acids are increased to be made available for energy, liver glycogen is

depleted in an aempt to maintain plasma glucose, and skeletal muscle

mass decreases as protein is metabolized to make amino acids available

for energy.

TMP14 pp. 994–995

36. C) The postabsorptive state begins approximately 2 hours after a meal

when plasma glucose has typically returned to normal. During the

postabsorptive state, counter regulatory mechanisms are activated

which help to maintain constant plasma glucose concentration. Thus,

insulin is reduced to decrease the cellular uptake of glucose and

glucagon is increased to promote hepatic production and release of

glucose. After several hours, both growth hormone and cortisol are also

increased to reduce skeletal muscle and adipose uptake of glucose. The

net effect of these mechanisms is to prevent hypoglycemia.

TMP14 pp. 986, 991–992, 994

37. B) hCG also binds to LH receptors on the interstitial cells of the testes

of the male fetus, resulting in the production of testosterone in male

fetuses up to the time of birth. This small secretion of testosterone is

what causes the fetus to develop male sex organs instead of female sex

organs.

TMP14 pp. 1033, 1060–1061

38. D) Insulin is a peptide hormone that is derived from proinsulin. It

binds to an enzyme linked receptor composed of 2 alpha and 2 beta

subunits, leading to an increase in tyrosine kinase activity.

TMP14 pp. 984–985

39. E) The exchange of calcium between the bone fluid compartment and

the ECF serves as a rapid and fast-acting mechanism to buffer changes

in extracellular fluid calcium concentration.

TMP14 p. 1005

40. B) In nonpregnant woman, the only significant source of estrogen is

ovarian follicles or corpus luteae. Menstruation begins when the corpus

luteum degenerates. Menstruation ends when developing follicles

secrete estrogen sufficiently to raise circulating concentration to a level

that stimulates regrowth of the endometrium.

TMP14 pp. 1039, 1042, 1046–1047

41. B) As a result of negative feedback, plasma levels of TSH are a sensitive

index of circulating levels of unbound (free) thyroid hormones. High

plasma levels of TSH indicate inappropriately low levels of free thyroid

hormones in the circulation, such as are present with autoimmune

destruction of the thyroid gland in persons with Hashimoto’s disease.

However, because elevated plasma levels of estrogen in pregnancy

increase hepatic production of TBG, the total amount (bound + free) of

thyroid hormones in the circulation is elevated. Plasma levels of thyroid

hormones are elevated in persons with Graves’ disease and in patients

with a pituitary TSH-secreting tumor, as well in patients given thyroid

extract for therapy.

TMP14 pp. 954, 958–962

42. F) The kidneys are essential for the conversion of inactive vitamin D

prohormones to the biologically active vitamin D hormone (1,25-

dihydroxycholecalciferol). This conversion is mediated by parathyroid

hormone acting in the proximal tubule epithelial cells. Therefore, with

impaired renal function, one would expect a decrease in plasma [1,25-

(OH)2D], along with a compensatory increase in PTH. The increased

plasma PTH causes bone resorption of calcium.

TMP14 p. 1015

43. D) The cells of the anterior pituitary that secrete LH and FSH, along

with the cells of the hypothalamus that secrete GnRH, are inhibited by

both estrogen and testosterone. The steroids taken by the woman caused

sufficient inhibition to result in cessation of the monthly menstrual

cycle.

TMP14 pp. 1033, 1047–1048

44. D) Patients with central diabetes insipidus have an inappropriately low

secretion rate of ADH in response to changes in plasma osmolality, but

their renal response to ADH is not impaired. Because plasma levels of

ADH are depressed, the ability to concentrate urine is impaired, and a

large volume of dilute urine is excreted. Loss of water tends to increase

plasma osmolality, which stimulates the thirst center and leads to a very

high rate of water turnover.

TMP14 p. 949

45. B) NO is the vasodilator that is normally released, causing vasodilation

in these arteries.

TMP14 pp. 1027, 1034

46. B) Hydroxyapatite is the major salt found in calcified bone, and the

osteon is composed of concentric layers of calcified bone. However, an

osteocyte is a quiescent cell that resides in lacunae (spaces). Osteoblasts

are the cells that actively form new bone.

TMP14 pp. 1003, 1005–1006

47. B) One of the major physiological roles for PTH is to promote the

conversion of 25-hydroxycholecalciferol, to the active 1,25-

dihydroxycholecalciferol in the proximal tubular epithelium. The other

choices represent normal physiological actions of PTH.

TMP14 pp. 1009–1012

48. D) Lethargy and myxedema are signs of hypothyroidism. Low plasma

levels of TSH indicate that the abnormality is in either the hypothalamus

or the pituitary gland. The responsiveness of the pituitary to the

administration of TRH suggests that pituitary function is normal and

that the hypothalamus is producing insufficient amounts of TRH.

TMP14 pp. 958–962

49. D) Inhibin prevents FSH release from the anterior pituitary,

preventing Sertoli cells from causing aromatization to produce estradiol.

TMP14 p. 1032

50. A) After menopause, the absence of feedback inhibition by estrogen

and progesterone results in extremely high rates of FSH secretion.

Women taking estrogen as part of hormone replacement therapy for

symptoms associated with postmenopausal conditions have suppressed

levels of FSH as a result of the inhibitory effect of estrogen.

TMP14 pp. 1050, 1051

51. D) Phosphodiesterase-5 receptors prevent hydrolysis of cyclic

guanosine monophosphate, thus keeping the levels high and

maintaining vasodilation.

TMP14 p. 1034

52. B) Glucagon stimulates glycogenolysis in the liver, but it has no

physiological effects in muscle. Both glucagon and cortisol increase

gluconeogenesis, and cortisol impairs glucose uptake by muscle.

TMP14 pp. 972–973, 992

53. C) Injection of insulin leads to a decrease in blood glucose

concentration. Hypoglycemia stimulates the secretion of GH, glucagon,

and epinephrine, all of which have counter regulatory effects to increase

glucose levels in the blood.

TMP14 pp. 945, 993–994

54. A) Prolonged fetal hypoxia during delivery can cause serious

depression of the respiratory center. Hypoxia may occur during delivery

because of compression of the umbilical cord, premature separation of

the placenta, excessive contraction of the uterus, or excessive anesthesia

of the mother.

TMP14 p. 1073

55. C) In general, peptide hormones are water soluble and are not highly

bound by plasma proteins. ADH, a neurohypophysial peptide hormone,

is virtually unbound by plasma proteins. In contrast, steroid and thyroid

hormones are highly bound to plasma proteins.

TMP14 pp. 929–930

56. C) The rise in intracellular calcium in the oocyte triggers the cortical

reaction in which granules that previously lay at the base of the plasma

membrane undergo exocytosis. This process leads to the release of

enzymes that “harden” the zona pellucida and prevent other sperm

from penetrating.

TMP14 p. 1025

57. B) Although estrogen and progesterone are essential for the physical

development of the breast during pregnancy, a specific effect of both

these hormones is to inhibit the actual secretion of milk. Even though

prolactin levels are increased 10- to 20-fold at the end of pregnancy, the

suppressive effects of estrogen and progesterone prevent milk

production until after the baby is born. Immediately after birth, the

sudden loss of both estrogen and progesterone secretion from the

placenta allows the lactogenic effect of prolactin to promote milk

production.

TMP14 pp. 1066–1067

58. C) The concentration of PTH strongly regulates the absorption of

calcium ion from the renal tubular fluid. A reduction in hormone

concentration reduces calcium reabsorption and increases the rate of

calcium excretion in the urine. The other choices either have lile effect

on or decrease calcium excretion.

TMP14 pp. 1011–1012

59. B) A pituitary tumor secreting increased amounts of TSH would be

expected to stimulate the thyroid gland to secrete increased amounts of

thyroid hormones. TSH stimulates several steps in the synthesis of

thyroid hormones, including the synthesis of thyroglobulin. Increased

heart rate is among the many physiological responses to high plasma

levels of thyroid hormones. However, high plasma levels of thyroid

hormones do not cause exophthalmos. Immunoglobulins cause

exophthalmos in Graves’ disease, the most common form of

hyperthyroidism.

TMP14 pp. 952, 957, 961

60. A) Hemorrhage decreases the activation of stretch receptors in the atria

and arterial baroreceptors. Decreased activation of these receptors

increases ADH secretion.

TMP14 p. 949

61. E) Choices A to D are true: LH secretion will be suppressed (B) by the

negative feedback effect of the estrogen from the tumor; consequently,

she will not have menstrual cycles (C), and because she will not have

normal cycles, no corpus luteae will develop, so no progesterone will be

formed (A). The high levels of estrogen produced by the tumor will

provide stimulation of osteoblastic activity to maintain normal bone

activity (D).

TMP14 pp. 1044, 1045

62. D) After eating a meal, insulin secretion is increased. As a result, there

is an increased rate of glucose uptake by both the liver and muscle.

Insulin also inhibits hormone-sensitive lipase, which decreases

hydrolysis of triglycerides in fat cells.

TMP14 pp. 985–987, 992

63. B) The primary function of testosterone in the embryonic development

of males is to stimulate formation of the male sex organs.

TMP14 pp. 219–220, 364, 383, 405, 949–950

64. B) FSH stimulates the production of estrogens from Sertolis cells in the

testis. The Sertoli cells receive testosterone from Leydig cells (stimulated

by LH) and use the testosterone to make estrogen.

TMP14 p. 1023

65. C) The reduction in hydrogen ion indicated by the elevation in pH

increases the concentration of negatively charged phosphate ion species

available for ionic combination with calcium ions. Consequently, the

free calcium ion concentration is reduced.

TMP14 pp. 1011–1012

66. A) Prostate fluid contains calcium, citrate, phosphate and fibrinolysin.

The function of prostate fluid is to help neutralize the acidic

environment associated with other seminal fluids and thus improve

sperm motility.

TMP14 p. 1024

67. C) During suckling, stimulation of receptors on the nipples increases

neural input to both the supraoptic and paraventricular nuclei.

Activation of these nuclei leads to the release of oxytocin and

neurophysin from secretion granules in the posterior pituitary gland.

Suckling does not stimulate the secretion of appreciable amounts of

ADH.

TMP14 pp. 1066, 1067

68. C) In Conn’s syndrome, large amounts of aldosterone are secreted.

Because aldosterone causes sodium retention, hypertension is a

common finding in patients with this condition. However, the degree of

sodium retention is modest, as is the resultant increase in extracellular

fluid volume. This occurs because the rise in arterial pressure offsets the

sodium-retaining effects of aldosterone, limiting sodium retention and

permiing daily sodium balance to be achieved.

TMP14 pp. 970, 981

69. C) The activity of stored sperm is aenuated as a result of the acidic

environment. After ejaculation, uterine and fallopian fluids wash away

inhibitory factors, allowing for full activation of the spermatozoa.

TMP14 pp. 1024–1025

70. D) DHEA sulfate produced by the fetal adrenal gland diffuses to the

placenta and is converted to DHEA and then to estradiol and provides

estradiol to the mother.

TMP14 pp. 1060, 1061

71. D) Sporadic nursing of the mother results in a lack of prolactin surge

because mechanosensors in the nipple cause prolactin release. Without

prolactin release, there is a lack of milk production, and the mother

eventually will not be able to provide milk for the baby.

TMP14 pp. 1066, 1067

72. A) Persons with Addison’s disease have diminished secretion of both

glucocorticoids (cortisol) and mineralocorticoids (aldosterone). In

persons with Cushing’s disease or Cushing’s syndrome, cortisol

secretion is elevated, but aldosterone secretion is normal. A low-sodium

diet is associated with a high rate of aldosterone secretion but a

secretion rate of cortisol that is normal. By inhibiting the generation of

angiotensin II and thus the stimulatory effects of angiotensin II on the

zona glomerulosa, administration of a converting enzyme inhibitor

would decrease aldosterone secretion without altering the rate of

cortisol secretion.

TMP14 pp. 971–972, 979–980

73. E) Spermatogonia undergo two rounds of meiotic division, leading to

the production of four haploid spermatids. The spermatids ultimately

differentiate into mature sperm.

TMP14 pp. 1021–1022

74. B) Progesterone is required to maintain the decidual cells of the

endometrium. If progesterone levels fall, as they do during the last days

of a nonpregnant menstrual cycle, menstruation will follow within a few

days, with loss of pregnancy. Administration of a compound that blocks

the progesterone receptor during the first few days after conception will

terminate the pregnancy.

TMP14 pp. 1060–1061

75. D) An inappropriately high rate of ADH secretion from the lung

promotes excess water reabsorption, which tends to produce

concentrated urine and a decrease in plasma osmolality. Low plasma

osmolality suppresses both thirst and ADH secretion from the pituitary

gland.

TMP14 pp. 404, 949

76. B) A very high plasma concentration of progesterone maintains the

uterine muscle in a quiescent state during pregnancy. In the final month

of gestation, the concentration of progesterone begins to decline,

increasing the excitability of the muscle.

TMP14 pp. 971–972, 1027

77. D) The corpus luteum is the only source of progesterone. If she is not

having menstrual cycles, no corpus luteum is present.

TMP14 p. 1048

78. C) FSH stimulates the granulosa cells of the follicle to secrete estrogen.

TMP14 pp. 1040, 1048

79. E) In response to increased blood levels of glucose, plasma insulin

concentration normally increases during the 60-minute period following

oral intake of glucose. In type 1 diabetes mellitus, insulin secretion is

depressed. In contrast, in type 2 diabetes mellitus, insulin resistance is a

common finding, and at least in the early stages of the disease, there is

an abnormally high rate of insulin secretion.

TMP14 pp. 995–998

80. D) In Cushing’s syndrome, high plasma levels of cortisol impair

glucose uptake in peripheral tissues, which tends to increase plasma

levels of glucose. As a result, the insulin response to oral intake of

glucose is enhanced.

TMP14 pp. 996–998

81. B) In general, protein hormones cause physiological effects by binding

to receptors on the cell membrane. However, of the four protein

hormones indicated, only insulin activates an enzyme-linked receptor.

Aldosterone is a steroid hormone and enters the cytoplasm of the cell

before binding to its receptor.

TMP14 p. 932

82. D) hCG is secreted from the trophoblast cells beginning shortly after

the blastocyst implants in the endometrium.

TMP14 pp. 1060–1061

83. B) Aortic pressure increases due to the increase in left ventricular

pressure. The increase in left atrial pressure causes the foramen ovale to

close. The ductus arteriosus also closes within a short time after birth.

TMP14 pp. 1073–1075

84. A) Somnolence is a common feature of hypothyroidism. Palpitations,

increased respiratory rate, increased cardiac output, and weight loss are

all associated with hyperthyroidism.

TMP14 pp. 957, 962–963

85. C) An infant born of a mother with untreated diabetes will have

considerable hypertrophy and hyperfunction of the islets of Langerhans

in the pancreas. As a consequence, the infant’s blood glucose

concentration may fall to lower than 20 mg/dl shortly after birth.

TMP14 pp. 1078–1079

86. B) If a successful fertilization event occurs, followed by implantation in

the uterine wall, trophoblasts produce and secrete human chorionic

gonadotropin, which maintains the corpus luteum and its production of

estrogen and progesterone. Eventually, hCG levels decline in association

with increased placental production of progesterone and estrogen.

TMP14 p. 1042

87. E) Choices A to D would not stimulate PTH secretion. An increase in

calcium concentration (A) suppresses PTH secretion; calcitonin has lile

to no effect on PTH secretion (B); acidosis would increase free calcium in

the extracellular fluid, thereby inhibiting PTH secretion (C); and PTHreleasing hormone does not exist (D).

TMP14 pp. 1001, 1011

88. C) Potassium is a potent stimulus for aldosterone secretion, as is

angiotensin II. Therefore, a patient consuming a high-potassium diet

would exhibit high circulating levels of aldosterone.

TMP14 p. 971

89. B) The decidua and trophoblasts provide the nutrition needed to

provide nourishment of the blastocyst.

TMP14 pp. 1057, 1060–1062

90. C) Steroid hormones are not stored to any appreciable extent in their

endocrine-producing glands. This is true for aldosterone, which is

produced in the adrenal cortex. In contrast, there are appreciable stores

of thyroid hormones and peptide hormones in their endocrineproducing glands.

TMP14 p. 928

91. C) 1,25-Dihydroxycholecalciferol is formed only in the renal cortex.

Extensive renal disease reduces the amount of cortical tissue,

eliminating the source of this active calcium regulating hormone.

TMP14 p. 1015

92. C) The placenta cannot produce androgens but can only produce

DHEA by removal of the sulfate from DHEAS produced in the fetal

adrenal glands.

TMP14 p. 1060

93. A) The secretory phase of the endometrial cycle aligns with the luteal

phase of the ovarian cycle. Progesterone levels peak during this phase

and promote the vascularization and thickening of the endometrial

lining. If a fertilization event and subsequent implantation does not

occur, the corpus luteum involutes causing progesterone levels to fall

and the endometrial lining to slough off during menstruation.

TMP14 pp. 1046–1047

94. D) Because iodine is needed to synthesize thyroid hormones, the

production of thyroid hormones is impaired if iodine is deficient. As a

result of feedback, plasma levels of TSH increase and stimulate the

follicular cells to increase the synthesis of thyroglobulin, which results

in a goiter. Increased metabolic rate, sweating, nervousness, and

tachycardia are all common features of hyperthyroidism, not

hypothyroidism, due to iodine deficiency.

TMP14 pp. 960–963

95. C) Because of the effects of thyroid hormones to increase metabolism in

tissues, tissues vasodilate, thus increasing blood flow and cardiac

output. All the other choices increase in response to high plasma levels

of thyroid hormones.

TMP14 pp. 956–957

96. B) Sperm cell motility decreases as pH is reduced below 6.8. At a pH of

4.5, sperm cell motility is significantly reduced. However, the buffering

effect of sodium bicarbonate in the prostatic fluid raises the pH

somewhat, allowing the sperm cells to regain some mobility.

TMP14 p. 1024

97. B) A protein meal stimulates all three hormones indicated.

TMP14 pp. 945, 991, 993

98. C) Testosterone secreted by the testes in response to LH inhibits

hypothalamic secretion of GnRH, thereby inhibiting anterior pituitary

secretion of LH and FSH. Taking large doses of testosterone-like steroids

also suppresses the secretion of GnRH and the pituitary gonadotropic

hormones, resulting in sterility.

TMP14 p. 1033

99. C) Steroids with potent glucocorticoid activity tend to increase plasma

glucose concentration. As a result, insulin secretion is stimulated.

Increased glucocorticoid activity also diminishes muscle protein.

Because of feedback, cortisone administration leads to a decrease in

adrenocorticotropic hormone secretion and therefore a decrease in

plasma cortisol concentration.

TMP14 pp. 972–973

100. C) Inhibin is the hormone that has a negative feedback on the anterior

pituitary to prevent FSH from being released. Inhibin is produced by the

granulosa cells in the ovary.

TMP14 pp. 1040–1041

101. A) An increase in the concentration of PTH results in the stimulation

of existing osteoclasts and, over longer periods, increases the number of

osteoclasts present in the bone.

TMP14 pp. 1010–1011

102. B) In general, peptide hormones produce biological effects by binding

to receptors on the cell membrane. Peptide hormones are stored in

secretion granules in their endocrine-producing cells and have relatively

short half-lives because they are not highly bound to plasma proteins.

Protein hormones often have a rapid onset of action because, unlike

steroid and thyroid hormones, protein synthesis is usually not a

prerequisite to produce biological effects.

TMP14 pp. 926, 929–932

103. D) A pituitary tumor secreting GH is likely to present as an increase in

pituitary gland size. The anabolic effects of excess GH secretion lead to

enlargement of the internal organs, including the kidneys. Because

acromegaly is the state of excess GH secretion after epiphyseal closure,

increased femur length does not occur.

TMP14 p. 947

104. A) GH and cortisol have opposite effects on protein synthesis in

muscle. GH is anabolic and promotes protein synthesis in most cells of

the body, whereas cortisol decreases protein synthesis in extrahepatic

cells, including muscle. Both hormones impair glucose uptake in

peripheral tissues and therefore tend to increase plasma glucose

concentration. Both hormones also mobilize triglycerides from fat stores.

TMP14 pp. 943–944, 972–973

105. B) If the mother has had adequate amounts of iron in her diet, the

infant’s liver usually has enough stored iron to form blood cells for 4 to

6 months after birth. However, if the mother had insufficient iron levels,

severe anemia may develop in the infant after about 3 months of life.

TMP14 pp. 1072, 1077

106. A) High plasma levels of steroids with glucocorticoid activity

suppress CRH and, consequently, ACTH secretion. Therefore, the

adrenal glands would actually atrophy with chronic cortisone treatment.

Increased plasma levels of glucocorticoids tend to cause sodium

retention and increase blood pressure. They also tend to increase plasma

levels of glucose and, consequently, stimulate insulin secretion and Cpeptide, which is part of the insulin prohormone.

TMP14 pp. 972–973, 976–977, 979–980

107. B) During the postovulatory phase of the cycle, there is a negative

feedback relationship between progesterone and estrogen and the

hypothalamic pituitary axis. Therefore, progesterone suppresses GnRH

release.

TMP14 pp. 1040–1042

108. C) SRY is the region on the Y chromosome that encodes a

transcription factor that causes differentiation of Sertoli cells from

precursors in testis. If SRY is not present, granulosa cells in the ovary are

produced.

TMP14 p. 1029

109. D) Fertilization of the ovum normally takes place in the ampulla of

one of the fallopian tubes.

TMP14 p. 1055

110. D) Because insulin secretion is deficient in persons with type 1

diabetes mellitus, there is increased (not decreased) release of glucose

from the liver. Low plasma levels of insulin also lead to a high rate of

lipolysis; increased plasma osmolality, hypovolemia, and acidosis are all

symptoms of uncontrolled type 1 diabetes mellitus.

TMP14 pp. 995–996

111. E) Under acute conditions, an increase in blood glucose concentration

will decrease GH secretion. GH secretion is characteristically elevated in

the chronic pathophysiological states of acromegaly and gigantism.

Deep sleep and exercise are stimuli that increase GH secretion.

TMP14 pp. 945–946

112. D) All the steroids listed include pregnenolone early in their

biosynthetic pathway. 1,25(OH)2D is derived from vitamin D and does

not include pregnenolone in its biosynthetic pathway.

TMP14 pp. 965–967, 1007–1008

113. D) Estrogen and, to a lesser extent, progesterone secreted by the

corpus luteum during the luteal phase have strong feedback effects on

the anterior pituitary gland to maintain low secretory rates of both FSH

and LH. In addition, the corpus luteum secretes inhibin, which inhibits

the secretion of FSH.

TMP14 p. 1042

114. D) Under chronic conditions, the effects of high plasma levels of

aldosterone to promote sodium reabsorption in the collecting tubules

are sustained. However, persistent sodium retention does not occur

because of concomitant changes that promote sodium excretion. These

changes include increased arterial pressure, increased plasma levels of

g p p

atrial natriuretic peptide, and decreased plasma angiotensin II

concentration.

TMP14 pp. 961, 981

115. B) For reasons that are not entirely clear, the negative feedback

regulation between estrogen and LH that occurs throughout the ovarian

cycle briefly changes to a positive feedback mechanism. This occurs late

in the follicular phase, just prior to ovulation, when LH promotes

estrogen production and estrogen feeds back to stimulate the further

release of LH. This underlies the surge in LH just before ovulation.

TMP14 pp. 1040–1042

116. B) Circulating levels of free T4

 exert biological effects and are

regulated by feedback inhibition of TSH secretion from the anterior

pituitary gland. Protein-bound T4

 is biologically inactive. Circulating T4

is highly bound to plasma proteins, especially to TBG, which increases

during pregnancy. An increase in TBG tends to decrease free T4

, which

then leads to an increase in TSH secretion, causing the thyroid to

increase thyroid hormone secretion. Increased secretion of thyroid

hormones persists until free T4

 returns to normal levels, at which time

there is no longer a stimulus for increased TSH secretion. Therefore, in a

chronic steady-state condition associated with elevated TBG, high

plasma total T4

 (bound and free) and normal plasma TSH levels would

be expected. In this pregnant patient, the normal levels of total T4

, along

with high plasma levels of TSH, would indicate an inappropriately low

plasma level of free T4

. Deficient thyroid hormone secretion in this

patient would be consistent with Hashimoto’s disease, the most

common form of hypothyroidism.

TMP14 pp. 954, 958–962

117. D) The motor neurons of the spinal cord of the thoracic and lumbar

regions are the sources of innervation for the skeletal muscles of the

perineum involved in ejaculation.

TMP14 pp. 1026, 1027

118. A) Trophoblasts invade the endometrial lining of the uterus and

provide nutrients to the growing blastocyst until the placenta if formed.

TMP14 pp. 1056–1057

119. B) Bone is deposited in proportion to the compressional load that the

bone must carry. Continual mechanical stress stimulates osteoblastic

deposition and calcification of bone.

TMP14 pp. 1006–1007

120. D) Prolactin is produced in the anterior pituitary from lactotrope cells

and is responsible for promoting milk production and secretion.

TMP14 pp. 1067–1068

121. B) In the absence of 11-β-hydroxysteroid dehydrogenase, renal

epithelial cells cannot convert cortisol to cortisone; therefore, cortisol

will bind to the mineralocorticoid receptor and mimic the actions of

excess aldosterone. Consequently, this would result in hypertension

associated with suppression of the renin-angiotensin-aldosterone

system, along with hypokalemia.

TMP14 pp. 968–970, 980–981

122. D) In target tissues, nuclear receptors for thyroid hormones have a

greater affinity for T3

 than for T4

. The secretion rate, plasma

concentration, half-life, and onset of action are all greater for T4

 than for

T3

.

TMP14 pp. 953–955

123. C) Blocking the action of FSH on the Sertoli cells of the seminiferous

tubules interrupts the production of sperm. Choice C is the only option

that is certain to provide sterility.

TMP14 p. 1033

124. C) Oxytocin is secreted from the posterior pituitary gland and carried

in the blood to the breast, where it causes the cells that surround the

outer walls of the alveoli and ductile system to contract. Contraction of

these cells raises the hydrostatic pressure of the milk in the ducts to 10 to

20 mm Hg. Consequently, milk flows from the nipple into the baby’s

mouth.

TMP14 pp. 1068–1069

125. B) Resulting from the growing fetal-placental unit, there is a large

increase in metabolic demand during a normal pregnancy. Given that

metabolic demand is the major determinant for cardiac output, the

increase in metabolic demand during pregnancy causes an increase in

cardiac output.

TMP14 p. 1062

126. F) Persons with Cushing’s disease have a high rate of cortisol

secretion, but aldosterone secretion is normal. High plasma levels of

cortisol tend to increase plasma glucose concentration by impairing

glucose uptake in peripheral tissues and by promoting gluconeogenesis.

However, at least in the early stages of Cushing’s disease, the tendency

for glucose concentration to increase appreciably is counteracted by

increased insulin secretion.

TMP14 pp. 972–973, 979–980

127. A) In healthy patients, the secretory rates of ACTH and cortisol are

low in the late evening but high in the early morning. In patients with

Cushing’s syndrome (adrenal adenoma) or in patients taking

dexamethasone, plasma levels of ACTH are very low and are certainly

not higher than normal early morning values. In patients with

Addison’s disease, plasma levels of ACTH are elevated as a result of

deficient adrenal secretion of cortisol. The secretion of ACTH and

cortisol would be expected to be normal in Conn’s syndrome.

TMP14 pp. 977–980

128. B) Exercise stimulates GH secretion. Hyperglycemia, somatomedin,

and the hypothalamic inhibitory hormone somatostatin all inhibit GH

secretion. GH secretion also decreases as persons age.

TMP14 p. 945

129. C) A low-sodium diet would stimulate aldosterone but not cortisol

secretion. Increased atrial stretch associated with volume expansion

would stimulate atrial natriuretic peptide secretion but would not be

expected during a low-sodium diet.

TMP14 pp. 364, 405, 971–972

130. A) Adrenal gland hypofunction with Addison’s disease is associated

with decreased secretion of both aldosterone and cortisol. In Cushing’s

disease and Cushing’s syndrome associated with an ectopic tumor, the

mineralocorticoid-hypertension induced by high plasma levels of

cortisol would suppress aldosterone secretion. Neither a high-sodium

diet nor administration of a converting enzyme inhibitor would affect

cortisol secretion.

TMP14 pp. 971-972, 979–980

131. B) Blood returning from the placenta through the umbilical vein

passes through the ductus venosus. The blood coming from the placenta

has the highest concentration of oxygen found in the fetus.

TMP14 p. 1074

132. B) Osteoporosis, hypertension, hirsutism, and hyperpigmentation are

all symptoms of Cushing’s syndrome associated with high plasma levels

of ACTH. If the high plasma ACTH levels were the result of either a

pituitary adenoma or an abnormally high rate of corticotropin-releasing

hormone secretion from the hypothalamus, the patient would likely

have an enlarged pituitary gland. In contrast, the pituitary gland would

not be enlarged if an ectopic tumor were secreting high levels of ACTH.

TMP14 pp. 979–980

133. B) Prolactin secretion is inhibited, not stimulated, by the hypothalamic

release of dopamine into the median eminence. GH is inhibited by the

hypothalamic-inhibiting hormone somatostatin. The secretion of LH,

TSH, and ACTH are all under the control of the releasing hormones

indicated.

TMP14 p. 942

134. B) Increased heart rate, increased respiratory rate, and decreased

cholesterol concentration are all responses to excess thyroid hormone.

TMP14 pp. 956–958

135. C) hCG is produced by syncytial trophoblasts from the growing

blastocyst. hCG is responsible for maintaining the corpus luteum which

produces estrogens and progesterone up through approximately 12

weeks’ gestation. After that time, the placenta makes enough estrogen

and progesterone to sustain the pregnancy.

TMP14 pp. 1059–1060

136. D) By age 45 years, only a few primordial follicles remain in the

ovaries to be stimulated by gonadotropic hormones, and the production

of estrogen decreases as the number of follicles approaches zero. When

estrogen production falls below a critical value, it can no longer inhibit

g p g

the production of gonadotropic hormones from the anterior pituitary.

FSH and LH are produced in large quantities, but as the remaining

follicles become atretic, production by the ovaries falls to zero.

TMP14 pp. 1050, 1051

137. D) The binding of insulin to its receptor activates tyrosine kinase,

resulting in metabolic events leading to increased synthesis of fats,

proteins, and glycogen. In contrast, gluconeogenesis is inhibited.

TMP14 pp. 984–989

138. C) The secretion of chemical messengers (neurohormones) from

neurons into the blood is referred to as neuroendocrine secretion. Thus,

in contrast to the local actions of neurotransmiers at nerve endings,

neurohormones circulate in the blood before producing biological

effects at target tissues. Oxytocin is synthesized from magnocellular

neurons whose cell bodies are located in the paraventricular and

supraoptic nuclei and whose nerve terminals terminate in the posterior

pituitary gland. Target tissues for circulating oxytocin are the breast and

uterus, where the hormone plays a role in lactation and parturition,

respectively.

TMP14 pp. 925, 948–950

139. C) The placenta is hypoxic under normal physiological conditions.

The diffusion of oxygen from the maternal circulation to the fetal

circulation is enhanced by the fact that fetal hemoglobin carries a greater

quantity of oxygen at a given blood Po2 than maternal hemoglobin. In

addition, the hemoglobin concentration is greater in the fetal circulation

than in the maternal circulation.

TMP14 p. 1058

140. B) Inhibition of the iodide pump decreases the synthesis of thyroid

hormones but does not impair the production of thyroglobulin by

follicular cells. Decreased plasma levels of thyroid hormones result in a

low metabolic rate and lead to an increase in TSH secretion. Increased

plasma levels of TSH stimulate the follicular cells to synthesize more

thyroglobulin. Nervousness is a symptom of hyperthyroidism and is not

caused by thyroid hormone deficiency.

TMP14 pp. 951–952, 956–960

141. D) As the blastocyst implants, the trophoblast cells invade the

decidua, digesting and imbibing it. The stored nutrients in the decidual

cells are used by the embryo for growth and development. During the

first week after implantation, this is the only means by which the

embryo can obtain nutrients. The embryo continues to obtain at least

some of its nutrition in this way for up to 8 weeks, although the placenta

begins to provide nutrition after about the 16th day beyond fertilization

(a lile more than 1 week after implantation).

TMP14 p. 1056

142. A) Both ADH and oxytocin are peptides containing nine amino acids.

Their chemical structures differ in only two amino acids.

TMP14 p. 949

143. A) Because glucocorticoids decrease the sensitivity of tissues to the

metabolic effects insulin, they would exacerbate diabetes.

Thiazolidinediones and weight loss increase insulin sensitivity.

Sulfonylureas increase insulin secretion. If weight loss and the

aforementioned drugs are ineffective, exogenous insulin may be used to

regulate blood glucose concentration.

TMP14 pp. 991, 996–997

144. C) In the early stages of type 2 diabetes, the tissues have a decreased

sensitivity to insulin. As a result, there is a tendency for plasma glucose

to increase, in part because decreased hepatic insulin sensitivity leads to

increased hepatic glucose output. Because of the tendency for plasma

glucose to increase, there is a compensatory increase in insulin secretion,

including C-peptide, which is part of the insulin prohormone.

Hypovolemia and increased production of ketone bodies, although

commonly associated with uncontrolled type 1 diabetes, are not

typically present in the early stages of type 2 diabetes.

TMP14 pp. 984, 994–998

145. C) One of the most characteristic findings in respiratory distress

syndrome is failure of the respiratory epithelium to secrete adequate

quantities of surfactant into the alveoli. Surfactant decreases the surface

tension of the alveolar fluid, allowing the alveoli to open easily during

inspiration. Without sufficient surfactant, the alveoli tend to collapse,

and there is a tendency to develop pulmonary edema.

TMP14 p. 1074

146. D) Several circulatory changes occur in the fetal circulation after birth.

These include the closing of physiological shunts. The ductus arteriosus

is a shunt that carries blood from the fetal pulmonary artery into the

descending aorta, thus bypassing the pulmonary circulation. At birth,

this shunt closes as systemic resistance increases, causing blood to flow

back into the pulmonary circulation through the shunt. Within hours of

birth, the walls of the ductus arteriosus close, and eventually the closing

becomes fibrous for permanent closure.

TMP14 pp. 1074–1075

147. C) The primary controllers of ACTH, GH, LH, and TSH secretion from

the pituitary gland are hypothalamic-releasing hormones. They are

secreted into the median eminence and subsequently flow into the

hypothalamic-hypophysial portal vessels before bathing the cells of the

anterior pituitary gland. Conversely, prolactin secretion from the

pituitary gland is influenced primarily by the hypothalamic-inhibiting

hormone dopamine. Consequently, obstruction of blood flow through

the portal vessels would lead to reduced secretion of ACTH, GH, LH,

and TSH but increased secretion of prolactin.

TMP14 p. 942

148. D) Osteoblasts secrete all of these except pyrophosphate. Secretions

(alkaline phosphatase) from osteoblasts neutralize pyrophosphate, an

inhibitor of hydroxyapatite crystallization. Neutralization of

pyrophosphate permits the precipitation of calcium salts into collagen

fibers.

TMP14 pp. 1004–1006

149. B) In primary hyperparathyroidism, high plasma levels of PTH

increase the formation of 1,25-(OH)2D3, which increases intestinal

absorption of calcium. This action of PTH, along with its effects to

increase bone resorption and renal calcium reabsorption, leads to

hypercalcemia. However, because of the high filtered load of calcium,

calcium is excreted in the urine. High plasma levels of PTH also

decrease phosphate reabsorption and increase urinary excretion, leading

to a fall in plasma phosphate concentration.

TMP14 pp. 1009–1012, 1014–1015

150. A) Gamma radiation destroys the cells undergoing the most rapid

rates of mitosis and meiosis, the germinal epithelium of the testes. The

man described is said to have normal testosterone levels, suggesting that

the secretory paerns of GnRH and LH are normal and that his

interstitial cells are functional. Because he is not producing sperm, the

levels of inhibin secreted by the Sertoli cells would be maximally

suppressed, and his levels of FSH would be strongly elevated.

TMP14 p. 1033

151. B) In this experiment, the size of the thyroid gland increased because

TSH causes hypertrophy and hyperplasia of its target gland and

increased secretion of thyroid hormones. Increased plasma levels of

thyroid hormones inhibit the secretion of TRH, which decreases

stimulation of the pituitary thyrotropes, resulting in a decrease in the

size of the pituitary gland. Higher plasma levels of thyroid hormones

also increase metabolic rate and decrease body weight.

TMP14 pp. 955–955, 960

152. C) In this experiment, the size of the pituitary and adrenal glands

increased because CRH stimulates the pituitary corticotropes to secrete

ACTH, which in turn stimulates the adrenals to secrete corticosterone

and cortisol. Higher plasma levels of cortisol increase protein

degradation and lipolysis and therefore decrease body weight.

TMP14 pp. 972–974, 976–977

153. C) At birth, the neonatal liver is not fully functional. Therefore, it does

not excrete bilirubin properly over the first several days of life. The

increased concentration of circulating bilirubin gives infants a yellow

pigmentation in the skin and eyes (jaundice).

TMP14 pp. 1076–1077


Unit XV: Sports Physiology

1. A Tour de France rider has the following values under resting

conditions:

Oxygen consumption = 250 ml O2

/min

Hemoglobin concentration = 15 g Hg/dl

Arterial partial pressure of oxygen (PO 2

) = 100 mm Hg

Mixed venous saturation = 75 percent

When exercising, he has the following values:

Oxygen consumption = 3000 ml O2

/min

Hemoglobin concentration = 15 g/dl

Arterial PO 2

 = 100 mm Hg

Mixed venous saturation = 25%

What is the absolute increase in cardiac output with exercise?

A) 5 l/min

B) 15 l/min

C) 25 l/min

D) 30 l/min

2. A female university student is comfortably running a 10K

race. At 5 miles, which set of values would best describe her

blood composition?

Arterial PO 2 Arterial PCO 2 Mixed Venous PO 2

A) ↑ ↑ ↓

B) ↑ ↑ ↔

C) ↑ ↓ ↔

D) ↑ ↔ ↓

E) ↑ ↔ ↑

F) ↔ ↔ ↔

G) ↓ ↑ ↓

H) ↓ ↓ ↓

I) ↓ ↑ ↔

3. Which statement about respiration in exercise is most

accurate?

A) Maximum oxygen consumption of a male marathon

runner is less than that of an untrained average man

B) Maximum oxygen consumption can be increased about

100% by training

C) Maximum oxygen diffusing capacity of a male

marathon runner is much greater than that of an

untrained average man

D) Blood levels of oxygen and carbon dioxide are

abnormal during exercise

4. Olympic athletes who run marathons or cross-country ski

have much higher maximum cardiac outputs than

nonathletes. Which statement about the hearts of these

athletes compared with nonathletes is most accurate?

A) Stroke volume in the Olympic athletes is about 5%

greater at rest

B) The percentage increase in heart rate during maximal

exercise is much greater in the Olympic athletes

C) Maximum cardiac output is only 3% to 4% greater in the

Olympic athletes

D) Resting heart rate in the Olympic athletes is

significantly higher

5. In athletes who use androgens to increase performance,

which of the following would most likely occur?

A) Decreased high-density blood lipoproteins

B) Decreased low-density blood lipoproteins

C) Increased testicular function

D) Decreased incidence of hypertension

6. A person living in Maine trains regularly to run 10K races

and continually finishes in the middle of the pack. What is

the physiological limitation that prevents this person from

improving?

A) Limited ability to increase pulmonary ventilation

B) Limited ability to use the oxygen delivered to the tissue

C) Limited ability to increase cardiac output

D) Limited ability to dissipate the heat generated with

exercise

E) Limited ability to convert glucose to adenosine

triphosphate (ATP)

7. If muscle strength is increased with resistive training, which

condition will most likely occur?

A) A decrease in the number of myofibrils

B) An increase in mitochondrial enzymes

C) A decrease in the components of the phosphagen

energy system

D) A decrease in stored triglycerides

8. Which of the following normally occurs during exercise at

aerobic levels?

A) Increased arterial PCO 2

B) Increased alveolar PO 2

C) Increased alveolar-arterial PO 2

 gradient

D) Decreased alveolar-arterial Po2

 gradient

9. A 40-year-old man is performing a maximum oxygen

consumption test. He is 4 minutes into this 15-minute test.

Which of the following would best describe his blood

composition?

Arterial Po2 Arterial Pco2 Mixed venous Po2

A) ↓ ↔ ↓

B) ↓ ↑ ↓

C) ↑ ↑ ↑

D) ↑ ↓ ↑

E) ↑ ↑ ↔

F) ↑ ↓ ↓

G) ↑ ↔ ↑

H) ↓ ↓ ↓

10. Which of the following physiologic responses to acute

exercise do not normally occur?

A) Decreased blood flow to bone and gastrointestinal tract.

B) Increased cardiac output.

C) Increased minute ventilation

D) Decrease in sympathetic stimulation

Answers

1. B) At rest:

Arterial content (Ca) = 15 × 1.34 = 20 ml O2

/100 ml blood at 100%

saturation

Venous content (Cv) = 20 × 0.75 = 15 ml O2

/100 ml blood

Arteriovenous O2

 difference = 5 ml O2

/100 ml blood

Answer:

VO 2

 = Q (ml/min) (Ca – Cv) 250 ml O2

/min = Q (5 ml O2

/100 ml

blood)

Q = 250 ml O2

/min ÷ 5 ml O2

/100 ml blood

Q = 5.0 l/min

Exercising:

Arterial content (Ca) = 15 × 1.34 = 20 ml O2

/100 ml blood

Venous content (Cv) = 20 × 0.25 = 5 ml O2

/100 ml blood

Arteriovenous O2

 difference = 15 ml O2

/100 ml blood

Answer:

VO 2

 = Q (ml/min) (Ca – Cv) 3000 ml O2

/min = Q (15 ml O2

/100 ml

blood)

Q = 3000 ml O2

/min ÷ 15 ml O2

/100 ml blood

Q = 20 l/min

The increase in VO 2

 is 20 l/min − 5 l/min = 15 l/min.

TMP14 pp. 256, 525, 1079–1081

2. D) With exercise, an increase in arterial PO 2

 occurs as a result of

beer ventilation/perfusion. Arterial PCO 2

 may be normal or

slightly decreased. Because of the increased metabolic rate, the

venous PO 2

 will decrease.

TMP14 pp. 1079–1081

3. C) During exercise, the maximum oxygen consumption of a male

marathon runner is much greater than that of an untrained

average man. However, athletic training increases the maximum

oxygen consumption by only about 10%. Therefore, the maximum

oxygen consumption in marathon runners is probably partly

genetically determined. These runners also have a large increase in

maximum oxygen diffusing capacity, and their blood levels of

oxygen and carbon dioxide remain relatively normal during

exercise.

TMP14 p. 1079

4. B) When comparing Olympic athletes and nonathletes, there are

several differences in the responses of the heart. Stroke volume is

much higher at rest in an Olympic athlete, and heart rate is much

lower. The heart rate can increase approximately 270% in an

Olympic athlete during maximal exercise, which is a much greater

percentage than occurs in a nonathlete. In addition, the maximal

increase in cardiac output is approximately 30% greater in the

Olympic athlete.

TMP14 pp. 1080–1081

5. A) Use of male sex hormones (androgens) or other anabolic

steroids to increase muscle strength may increase athletic

performances under some conditions but can have adverse effects

on the body. Anabolic steroids increase the risk of cardiovascular

damage because they increase the prevalence of hypertension,

decrease high-density blood lipoproteins, and increase lowdensity blood lipoproteins. These factors all promote heart aacks

and strokes. These androgenic substances also decrease testicular

function, which decreases the formation of sperm and the body’s

own production of natural testosterone.

TMP14 p. 1083

6. C) Pulmonary ventilation is not a limitation because people

normally overventilate during exercise, and there are minimal

changes in arterial blood gases. The muscles use the oxygen

delivered to them. The limitation is the delivery of oxygen and

nutrients to muscle based on the limitation of an increase in

cardiac output. Increasing cardiac output increases exercise

performance. Under hot conditions, heat dissipation can limit

exercise performance. Muscles have minimal to no limitation in

converting glucose to ATP.

TMP14 pp. 1078–1081

7. B) During resistive training, the muscles that are contracted with

at least a 50% maximal force for at least three times a week

experience an optimal increase in muscle strength. This increase in

strength causes muscle hypertrophy, and several changes occur.

There will be an increase in the number of myofibrils and up to a

120% increase in mitochondrial enzymes. As much as a 60% to

80% increase in the components of the phosphagen energy system

can occur, and up to a 50% increase in stored glycogen can occur.

Also, as much as a 75% to 100% increase in stored triglycerides can

occur.

TMP14 p. 1078

8. C) During exercise at aerobic levels, the increase in respiration

occurs to maintain a normal alveolar Po2

, maybe a slight increase.

There is no increase in arterial Pco2

. With normal ventilation, there

would be no decrease in the alveolar–arterial Po2

 gradient, which

would actually decrease oxygen diffusion from the alveoli into the

blood. With a decreased venous Po2 there will be an increased

alveolar-arterial Po2 gradient.

TMP14 pp. 517, 1080

9. F) Arterial Po2

 will increase because of a beer V/Q match. Arterial

Pco2

 will be normal or decreased. Mixed venous Po2

 will decrease.

TMP14 pp. 1079–1080

10. D) During exercise, there are increases in blood flow to muscle

and decrease in blood flow to nonessential tissues. This is

accomplished by an increase in sympathetic stimulation. There are

also increases in cardiac output, ventilation, and cardiac

contractility.

TMP14 pp. 260, 796, 1079–1081


Normal Values for Selected

Common Laboratory

Measurements

Substance

Average

(“Normal”

Value)

Range Comment/Unit of Measure

Electrolytes

Sodium (Na

+

)

142 mmol/l 135-145

mmol/l

mmol/l = millimoles per liter

Potassium (K+

)

4.2 mmol/l 3.5-5.3

mmol/l

Chloride (Cl

)

106 mmol/l 98-108

mmol/l

Anion gap

12 mEq/l 7-16

mEq/l

mEq/l = milliequivalents per liter

Anion gap = Na

+ – Cl

– – HCO3−

Bicarbonate (HCO3

)

24 mmol/l 22-29

mmol/l

Hydrogen ion (H+

)

40 nmol/l 30-50

nmol/l

nmol/l = nanomoles per liter

pH, arterial

7.4 7.25-7.45

pH, venous

7.37 7.32-7.42

Calcium ion (Ca

2+

)

5.0 mg/dl 4.65-5.28

mg/dl

mg/dl = milligrams/deciliter

Average normal value can also be

expressed as ∼ 1.2 mmol/l or 2.4 mEq/l

Calcium, total

10.0 mg/dl 8.5-10.5

mg/dl

Magnesium ion (Mg

2+

)

0.8 mEq/l 0.6-1.1

mEq/l

Magnesium, total

1.8 mEq/l 1.3-2.4

mEq/l

Phosphate, total

3.5 mg/dl 2.5-4.5

mg/dl

In plasma, HPO4

=

is ∼1.05 mmol/l and

H2PO4

is 0.26 mmol/l

Nonelectrolyte Blood Chemistries

Albumin

4.5 g/dl 3.5-5.5

g/dl

g/dl = grams per deciliter

Alkaline phosphatase

M: 38-

126 U/l

U/l = units per liter

Substance

Average

(“Normal”

Value)

Range Comment/Unit of Measure

F: 70-230

U/l

Bilirubin, total

0.2-1.0

mg/dl

Bilirubin, conjugated

0-0.2

mg/dl

Blood urea nitrogen

(BUN)

14 mg/dl 10-26

mg/dl

Creatinine

1.0 mg/dl 0.6-1.3

mg/dl

Varies depending on muscle mass, age, and

sex

Glucose

90 mg/dl 70-115

mg/dl

Osmolarity

282 mOsm/l 275-300

mOsm/l

mOsm/l = milliosmoles per liter

Osmolality is expresses as mOsm/kg of

water

Protein, total

7.0 g/dl 6.0-8.0

g/dl

Uric acid

M: 3.0-

7.4

mg/dl

F: 2.1-6.3

mg/dl

Blood Gases

O2 saturation, arterial

98% 95%-99% Percentage of hemoglobin molecules

saturated with oxygen

PO2

, arterial

90 mm Hg 80-100

mm Hg

PO2 = partial pressure of oxygen in

millimeters of mercury

PO2

, venous

40 mm Hg 25-40

mm Hg

PCO2

, arterial

40 mm Hg 35-45

mm Hg

PCO2 = partial pressure of carbon dioxide in

millimeters of mercury

PCO2

, venous

45 mm Hg 41-51

mm Hg

Hematology

Substance

Average

(“Normal”

Value)

Range Comment/Unit of Measure

Hematocrit (Hct)

M: 42% M:

39%-49%

F: 38% F:

35%-45%

Hemoglobin (Hgb)

M: 15 g/dl M: 13.5-

17.5 g/dl

F: 14g/dl F: 12-16

g/dl

Red blood cells (RBCs)

M: 5.5 × 10

8

/µl 4.3-5.7 ×

10

8

/µl

Number of cells per microliter of blood

F: 4.7 × 10

8

/µl 4.3-5.7 ×

10

8

/µl

Mean corpuscular

(RBC) volume (MCV)

90 fl 80-100 fl fl = femtoliters

Prothrombin time (PT)

10-14

seconds

Time required for the plasma to clot during a

special test

Platelets

150-450

× 10

3

/µl

White blood cells, total

4.5-11.0

× 10

3

/µl

Neutrophils

57%-67% Percentage of total white blood cells

Lymphocytes

23%-33% Percentage of total white blood cells

Monocytes

3%-7% Percentage of total white blood cells

Eosinophils

1%-3% Percentage of total white blood cells

Basophils

0%-1% Percentage of total white blood cells

Lipids

Total cholesterol

<200

mg/dl

Substance

Average

(“Normal”

Value)

Range Comment/Unit of Measure

Low-density

lipoprotein (LDL)

<130

mg/dl

High-density

lipoprotein (HDL)

M: >29

mg/dl

F: >35

mg/dl

Triglycerides

M: 40-

160

mg/dl

F: 35-135

mg/dl

This table is not an exhaustive list of common laboratory values. Most of these values are

approximate reference values used by the University of Mississippi Medical Center Clinical

Laboratories; normal ranges may vary among different clinical laboratories. Average

“normal” values and units of measure may also differ slightly from those cited in the Guyton

and Hall Textbook of Medical Physiology, 14th edition. For example, electrolytes are often

reported in milliequivalents per liter (mEq/l), a measure of electrical charge of an electrolyte,

or in millimoles per liter (mmol/l).



















guyton and hall physiology review

guyton and hall physiology

guyton and hall physiology review mcq

guyton and hall physiology short notes

guyton and hall physiology review latest edition

guyton and hall physiology review questions pdf

guyton and hall physiology mcqs

guyton and hall physiology review 4th edition

guyton and hall physiology lectures


guyton and hall physiology ppt

guyton and hall physiology anki

guyton and hall physiology anki deck

guyton and hall physiology audiobook

guyton and hall textbook of medical physiology apa citation

guyton and hall medical physiology 13th edition amazon

guyton and hall textbook of medical physiology south asian edition pdf download

guyton and hall textbook of medical physiology 13th edition amazon

guyton and hall textbook of medical physiology 14th edition amazon

guyton and hall physiology south asia edition pdf download

guyton and hall physiology south asia edition pdf

guyton and hall physiology book price

guyton and hall physiology book price in pakistan

guyton and hall physiology book

guyton and hall medical physiology book

guyton hall physiology question book

guyton and hall textbook of medical physiology e-book

physiology by guyton and hall

medical physiology by guyton and hall

guyton and hall textbook of medical physiology book buy

guyton and hall medical physiology review book

guyton and hall physiology chapters

guyton and hall physiology contents

guyton and hall physiology chapter 1

guyton and hall physiology citation

guyton and hall physiology table of contents

guyton and hall textbook of medical physiology citation

guyton and hall textbook of medical physiology chapters

guyton and hall textbook of medical physiology cite

guyton and hall textbook of medical physiology citation apa

guyton and hall textbook of medical physiology content

guyton and hall textbook of medical physiology deutsch

guyton and hall textbook of medical physiology 14th ed

guyton and hall textbook of medical physiology 12th ed citation

guyton and hall medical physiology ebook

guyton and hall human physiology

guyton and hall physiology kit

guyton and hall physiology lecture ppt

guyton and hall physiology latest

guyton and hall textbook of medical physiology latest international edition

guyton and hall physiology medical

guyton and hall medical physiology 14th

guyton and hall medical physiology notes

guyton and hall medical physiology review

guyton and hall textbook of medical physiology 14th edition citation

guyton and hall textbook of medical physiology 12th ed

guyton and hall physiology notes

guyton and hall textbook of medical physiology online

guyton and hall of medical physiology

guyton and hall textbook of medical physiology 14th

guyton and hall textbook of medical physiology 12th edition citation

guyton and hall physiology price

guyton and hall textbook of medical physiology price

guyton and hall textbook of medical physiology ppt

guyton and hall physiology questions

guyton and hall physiology quizlet

guyton and hall physiology review questions

guyton and hall textbook of medical physiology quizlet

guyton and hall textbook of medical physiology quiz

guyton and hall physiology review questions 4th edition pdf

guyton and hall physiology review questions 4th edition

guyton and hall textbook of medical physiology quora

guyton and hall physiology review 3rd edition

guyton and hall physiology review 2nd edition

guyton and hall physiology south asia edition

guyton and hall physiology summary

guyton and hall textbook of medical physiology summary

guyton and hall physiology third south asia edition pdf download

guyton and hall textbook of medical physiology south asian edition

guyton and hall textbook of medical physiology 2nd south asia edition pdf

guyton and hall physiology textbook

guyton and hall medical physiology textbook

guyton and hall physiology videos

guyton and hall textbook of medical physiology videos

guyton and hall textbook of medical physiology tiếng việt

guyton and hall physiology x ray

guyton and hall physiology xr

guyton and hall physiology youtube

guyton and hall physiology zotero

guyton and hall physiology zone

guyton and hall physiology zuid korea

can guyton and hall physiology exam

can guyton and hall physiology edition

can guyton and hall physiology essay

can guyton and hall physiology enzymes

can guyton and hall physiology journal

can guyton and hall physiology jobs

can guyton and hall physiology journal impact factor

can guyton and hall physiology kit

can guyton and hall physiology keys

can guyton and hall physiology of arts

can guyton and hall physiology of medicine

guyton and hall renal physiology

guyton and hall respiratory physiology

can guyton and hall physiology wikipedia

can guyton and hall physiology wiki

can guyton and hall physiology workshop

can guyton and hall physiology x ray

can guyton and hall physiology xr

can guyton and hall physiology youtube

can guyton and hall physiology yacht

can guyton and hall physiology zone

can guyton and hall physiology zurich

can guyton and hall physiology zotero

how guyton and hall physiology essay

how guyton and hall physiology exam

how guyton and hall physiology edition

how guyton and hall physiology endnote

how guyton and hall physiology journal

how guyton and hall physiology journal impact factor

how guyton and hall physiology jobs

how guyton and hall physiology kit

how guyton and hall physiology key

how guyton and hall physiology keys

how guyton and hall physiology of arts

how guyton and hall physiology of medicine

how guyton and hall physiology wikipedia

how guyton and hall physiology wiki

how guyton and hall physiology workshop

how guyton and hall physiology x ray

how guyton and hall physiology youtube

how guyton and hall physiology yacht

how guyton and hall physiology zone

how guyton and hall physiology zurich

how guyton and hall physiology zotero

how guyton and hall physiology zones

introduction to physiology

human physiology

الفيزيولوجيا الطبية

محاضرات physiology

which guyton and hall physiology edition

which guyton and hall physiology exam

which guyton and hall physiology essay

which guyton and hall physiology journal

which guyton and hall physiology jobs

which guyton and hall physiology journal impact factor

which guyton and hall physiology kit

which guyton and hall physiology key

which guyton and hall physiology keys

which guyton and hall physiology of medicine

which guyton and hall physiology of arts


F, Female; M, Male. 

Comments

Search This Blog

Archive

Show more

Popular posts from this blog

TRIPASS XR تري باس

CELEPHI 200 MG, Gélule

ZENOXIA 15 MG, Comprimé

VOXCIB 200 MG, Gélule

Kana Brax Laberax

فومي كايند

بعض الادويه نجد رموز عليها مثل IR ، MR, XR, CR, SR , DS ماذا تعني هذه الرموز

NIFLURIL 700 MG, Suppositoire adulte

Antifongiques مضادات الفطريات

Popular posts from this blog

علاقة البيبي بالفراولة بالالفا فيتو بروتين

التغيرات الخمس التي تحدث للجسم عند المشي

إحصائيات سنة 2020 | تعداد سكَان دول إفريقيا تنازليا :

ما هو الليمونير للأسنان ؟

ACUPAN 20 MG, Solution injectable

CELEPHI 200 MG, Gélule

الام الظهر

VOXCIB 200 MG, Gélule

ميبستان

Popular posts from this blog

TRIPASS XR تري باس

CELEPHI 200 MG, Gélule

Popular posts from this blog

TRIPASS XR تري باس

CELEPHI 200 MG, Gélule

ZENOXIA 15 MG, Comprimé

VOXCIB 200 MG, Gélule

Kana Brax Laberax

فومي كايند

بعض الادويه نجد رموز عليها مثل IR ، MR, XR, CR, SR , DS ماذا تعني هذه الرموز

NIFLURIL 700 MG, Suppositoire adulte

Antifongiques مضادات الفطريات

Popular posts from this blog

Kana Brax Laberax

TRIPASS XR تري باس

PARANTAL 100 MG, Suppositoire بارانتال 100 مجم تحاميل

الكبد الدهني Fatty Liver

الم اسفل الظهر (الحاد) الذي يظهر بشكل مفاجئ bal-agrisi

SEDALGIC 37.5 MG / 325 MG, Comprimé pelliculé [P] سيدالجيك 37.5 مجم / 325 مجم ، قرص مغلف [P]

نمـو الدمـاغ والتطـور العقـلي لـدى الطفـل

CELEPHI 200 MG, Gélule

أخطر أنواع المخدرات فى العالم و الشرق الاوسط

Archive

Show more